Sie sind auf Seite 1von 98

Login ID:1363169/Student Name:SSN/Overall Score:147

Student ScoreCard
SimCET-4

Score:147

Percentile:100

Overall: Overall

Score:147

Percentile:100

2)

3)

4)

5)

Pr

es

1)

iv
at
e

Li
m

ite
d

Question: 1
In 4 of the 5 figures given below, the left hand side bears a certain relationship with the right hand side. Choose the one where such a
relationship does not exist.

ou

rc

Explanation:
To form RHS, a top element and bottom element from LHS are replaced by their water images and the middle figure is replaced by its
mirror image, only in figure [2] this relation is not present. Hence, [2].

2)

3)

4)

5)

Le

ar

1)

ni

ng

es

Question: 2

IM

Explanation:
To form RHS, elements in LHS invert vertically and one line gets reduced in topelement. Only [3] is not following the pattern. Hence, [3].
Question: 3

IMS Learning Resources Pvt.Ltd.,Mumbai.All copyrights to this material vestswith IMS Learning Resources Pvt.Ltd.
No part of this materials either in part oras a whole shall be copied,printed,electronically reproduced,sold or distributed without the written
consent of IMS Learing Resources Pvt.Ltd.and any such violation would entail initiation of suitable legal proceedings.

Copyright

2)

3)

4)

5)

Li
m

1)

ite
d

Login ID:1363169/Student Name:SSN/Overall Score:147

iv
at
e

Explanation:
To form RHS, elements in LHS rotate by 90 in clockwise direction. Only [3] is not following the pattern. Hence, [3].

3)

4)

5)

rc

ou

2)

es

1)

es

Pr

Question: 4

ng

Explanation:
Top element of RHS is a water image of the left-hand side element and bottom elementof RHS is a mirror image of the left-hand side
element. In option [5], this relationis not seen. Hence, [5].

IM

Le

ar

ni

Question: 5

1)

2)

3)

4)

5)

Explanation:

IMS Learning Resources Pvt.Ltd.,Mumbai.All copyrights to this material vestswith IMS Learning Resources Pvt.Ltd.
No part of this materials either in part oras a whole shall be copied,printed,electronically reproduced,sold or distributed without the written
consent of IMS Learing Resources Pvt.Ltd.and any such violation would entail initiation of suitable legal proceedings.

Copyright

Login ID:1363169/Student Name:SSN/Overall Score:147

To form RHS, a black spot in the top element and in bottom element of LHS movesone step ahead in anticlockwise direction and in middle
element it moves one stepahead in clockwise direction. This relation is not present in option [4]. Hence, [4].

Ascend

2)

Enervate

3)

Habituate

4)

Claim

5)

Fade

Li
m

1)

ite
d

Question: 6
Choose the correct synonyms from the given options.
Acclimatize

iv
at
e

Explanation:
Acclimatize means to adapt (oneself), especially to environmental or climatic changes. Thus, habituate is the correct synonym. Hence,
[3].

2)

Frill

3)

Shroud

4)

Support

5)

Visage

es

Acclaim

rc

1)

Pr

Question: 7
Countenance

ou

Explanation:
Visage is the synonym of countenance which means the face or facial expression of a person. Hence, [5].

3)

Deceiving

4)

Pampering

5)

Spoiling

Offending

ng

2)

ni

Delaying

ar

1)

es

Question: 8
Procrastinating

IM

Question: 9
Remonstrate

Le

Explanation:
Procrastinating means to postpone or delay needlessly. Hence, [1].

1)

Appraise

2)

Depress

3)

Exalt

4)

Reproach

5)

Reclaim

Explanation:

IMS Learning Resources Pvt.Ltd.,Mumbai.All copyrights to this material vestswith IMS Learning Resources Pvt.Ltd.
No part of this materials either in part oras a whole shall be copied,printed,electronically reproduced,sold or distributed without the written
consent of IMS Learing Resources Pvt.Ltd.and any such violation would entail initiation of suitable legal proceedings.

Copyright

Login ID:1363169/Student Name:SSN/Overall Score:147

Remonstrate means to say or plead in protest. Reproach (which means to express disapproval of) is the correct synonym. Hence, [4].

Calculation

2)

Guesswork

3)

Certainty

4)

Genuineness

5)

Vocation

Explanation:
Surmise means guess or conjecture. So the synonym of surmise is guesswork. Hence, [2].

iv
at
e

Question: 11
Refer to the data below and answer the questions that follow.
What is the measure of angle PQR (in degrees)?
30

2)

45

3)

60

4)

90

5)

Cannot be determined

es

Pr

1)

Li
m

1)

ite
d

Question: 10
Surmise

Question: 12
What is the measure of angle PRS (in degrees)?

es

ou

rc

Explanation:
Since PR is the diameter of the circle, angles PQR and PSR are 90 degrees.
In triangle PRS, angle PSR = 90 degrees and angle RPS = 30 degrees. Therefore angle PRS = 180 90 30 = 60 degrees.
Hence both the questions can be answered.
Hence, [4].

30

2)

45

3)

60

4)

90

5)

Cannot be determined

Le

ar

ni

ng

1)

IM

Explanation:
Since PR is the diameter of the circle, angles PQR and PSR are 90 degrees.
In triangle PRS, angle PSR = 90 degrees and angle RPS = 30 degrees. Therefore angle PRS = 180 90 30 = 60 degrees.
Hence both the questions can be answered.
Hence, [3].
Question: 13
Choose the correct alternative.
The retailer has two washing machines of cost pricesRs. 6000 andRs. 15000. He sold first machine at 20% loss and second machine at
8% profit. Find his overall profit/loss in the transaction.

IMS Learning Resources Pvt.Ltd.,Mumbai.All copyrights to this material vestswith IMS Learning Resources Pvt.Ltd.
No part of this materials either in part oras a whole shall be copied,printed,electronically reproduced,sold or distributed without the written
consent of IMS Learing Resources Pvt.Ltd.and any such violation would entail initiation of suitable legal proceedings.

Copyright

Login ID:1363169/Student Name:SSN/Overall Score:147

1)

12% loss

2)

4% loss

3)

4% profit

4)

2% profit

5)

No profit/loss

iv
at
e

Li
m

ite
d

Explanation:

3)

51

4)

61

5)

71

rc

41

ou

2)

es

31

1)

es

Pr

Question: 14
For Independence Day celebrations, students of a class are made to stand in lines having equal number of students in each line. If each
line contains 2 or 3 or 4 or 5 or 6 students, in each case, there is one excess student who does not get any line to stand in. What is the
minimum number of students in the class?

ni

ng

Explanation:
The minimum number of students in the class is LCM of (2, 3, 4, 5 and 6) + 1.
LCM of 2,3,4,5 and 6 = 60. Therefore, the minimum number of students in the class = 60 + 1 = 61. Hence, [4].

2)

Rs. 900

3)

Rs. 960

4)

ar

Rs. 840

Le

1)

Question: 15
If certain amount yields a simple interest of Rs. 720 in 3 years, how much interest will it yield in 4 years at the same rate of interest?

IM

Rs. 1200

5)

Cannot be determined

Explanation:

IMS Learning Resources Pvt.Ltd.,Mumbai.All copyrights to this material vestswith IMS Learning Resources Pvt.Ltd.
No part of this materials either in part oras a whole shall be copied,printed,electronically reproduced,sold or distributed without the written
consent of IMS Learing Resources Pvt.Ltd.and any such violation would entail initiation of suitable legal proceedings.

Copyright

Login ID:1363169/Student Name:SSN/Overall Score:147

ite
d

Question: 16
Refer to the data below and answer the questions that follow.

ou

rc

es

Pr

iv
at
e

Li
m

Following pie-chart shows the break-up of the expenditure incurred by an automobile company on salaries of employees in different
departments. Study the pie chart and answer the questions that follow.

3)

Rs. 36 Crores

4)

Rs. 28.8 Crores

5)

None of these

Rs. 129.6 Crores

ng

2)

ni

Rs. 43.2 Crores

ar

1)

es

If the total expenditure of the company on salaries of all employees isRs. 360 Crores, then find the total expenditure on salary of Quality
Control department.

IM

Le

Explanation:

Question: 17
Calculate the measure of the central angle of the sector for the salary of Maintenance department.
1)

36 degrees

2)

28.8 degrees

3)

129.6 degrees

IMS Learning Resources Pvt.Ltd.,Mumbai.All copyrights to this material vestswith IMS Learning Resources Pvt.Ltd.
No part of this materials either in part oras a whole shall be copied,printed,electronically reproduced,sold or distributed without the written
consent of IMS Learing Resources Pvt.Ltd.and any such violation would entail initiation of suitable legal proceedings.

Copyright

Login ID:1363169/Student Name:SSN/Overall Score:147

4)

72 degrees

5)

None of these

Explanation:

2)

Rs. 60 Crores

3)

Rs. 90 Crores

4)

Rs. 120 Crores

5)

None of these

Li
m

Rs. 45 Crores

iv
at
e

1)

ite
d

Question: 18
If the total salary expenditure on Maintenance department isRs. 30 Crores, how much will be the salary expenditure on Design
department?

es

Pr

Explanation:
Percent salary expenditure on maintenance = 8% and percent salary expenditure on design = 24%. Therefore the salary expenditure on
design is three times the salary expenditure on maintenance. Therefore the salary expenditure on design department = 3 30 = 90
Crores. Hence, [3].

ou

rc

Question: 19
If the total expenditure on salary of the company isRs. 450 Crores, by what percentage is the salary expenditure on the Production
department more than the salary expenditure on Design department?
10%

2)

20%

3)

40%

4)

50%

5)

None of these

ng

es

1)

IM

Le

ar

ni

Explanation:

Question: 20
If in the next year, the company increases the salary expenditure on Maintenance department by 25% due to addition of some new
employees, while the salary expenditure on other departments remains the same, what is the percentage increase in the salary
expenditure of the company next year with respect to this year?
1)

5%

IMS Learning Resources Pvt.Ltd.,Mumbai.All copyrights to this material vestswith IMS Learning Resources Pvt.Ltd.
No part of this materials either in part oras a whole shall be copied,printed,electronically reproduced,sold or distributed without the written
consent of IMS Learing Resources Pvt.Ltd.and any such violation would entail initiation of suitable legal proceedings.

Copyright

Login ID:1363169/Student Name:SSN/Overall Score:147

2)

10%

3)

20%

4)

2%

5)

Cannot be determined

Li
m

ite
d

Explanation:
Suppose the total salary expenditure of the company is Rs. 100x, out of which salary expenditure on maintenance department is Rs. 8x. If
there is an increase of 25% next year, the salary expenditure on maintenance department next year will be 8x 1.25 = 10x (or addition of
2x). If the salary expenditure on all other departments remains unchanged, total addition to salary expenditure next year = Rs. 2x.
Therefore the percent increase in the total salary expenditure next year = 2%.
Hence, [4].
Question: 21
Read the following passage carefully and answer the questions given below it.

es

Pr

iv
at
e

The collision of four fundamental economic forces urbanization, technology, demographics and globalization is producing monumental
change. Global competition and technological change have sped up creative destruction and outpaced the ability of labour markets to
adapt. Job creation is a critical challenge for most policy makers even as businesses complain about critical skill gaps. Greying
populations are starting to fray social safety nets, and for debt-ridden societies in advanced economies, the challenge can only get more
pressing as the cost of capital starts to rise. Much-needed productivity growth continues to elude the public sector. Income inequality is
rising and causing a backlash, in some cases, targeted at the very interconnections of trade, finance and people that have fuelled the
growth of the past three decades.

es

ou

rc

Just as many businesses are being forced to reassess their strategy and reimagine their assumptions, the government must do the same.
The political-leadership challenge triggered by these trend breaks is made even more urgent by the growing number of outlets for public
expression and participation. Citizens around the world demand that governments deliver public services in shorter time frames, of
consistent quality, and often at lower cost. In times of tightening budgets, short election cycles and instant feedback loops, the room for
error by public-sector leaders is small. From Brazil to Egypt to Hong Kong to Ukraine, it is common to see large groups of citizens taking
to the streets, impatient for change.

1)

Governments have been subject to unforeseen political and economic forces.

2)

Governments all over the world have become infamous for their inertia.

3)

Le

ar

ni

ng

For public-sector officials, often the challenge isnt lack of vision but short time frames, competing priorities, and flawed delivery. Many
governments have risen to the occasion. An Asian country reduced street crime by 35 percent in the first year of a transformation
program. A South American government reduced hospital waiting lists by 80 percent and increased by more than 50 percent the number
of top graduates choosing to become teachers. An emerging-market government introduced a social-security scheme to hundreds of
thousands of workers in two months. In each of these cases, policy makers used what McKinsey calls a Delivery 2.0 approach a welldesigned programme with appropriate metrics, experimental delivery labs, small and high-powered execution teams, visible support from
leaders, and a culture of performance accountability.
Which of the following can be inferred from the first paragraph?

IM

The increasing populations of many countries have put a strain on social safety nets.

4)

Countries are grappling with their inability to match skill requirements.

5)

The current wave of change may wipe out weaker economies in the long run.

Explanation:
The first paragraph mentions that job creation is a challenge for most policy makers and that businesses increasingly face critical skill
gaps. It can be inferred that countries are unable to meet the skill requirements in businesses. [1] can be negated as political forces are
not mentioned. [2] cannot be inferred as it is a generalization. [3] is incorrect, as the paragraph mentions greying populations (i.e.
increasing proportion of elderly people), not increasing populations. There is no basis to infer [5] in the first paragraph. Hence, [4].

IMS Learning Resources Pvt.Ltd.,Mumbai.All copyrights to this material vestswith IMS Learning Resources Pvt.Ltd.
No part of this materials either in part oras a whole shall be copied,printed,electronically reproduced,sold or distributed without the written
consent of IMS Learing Resources Pvt.Ltd.and any such violation would entail initiation of suitable legal proceedings.

Copyright

Login ID:1363169/Student Name:SSN/Overall Score:147

Question: 22
Which of the following statements is true, according to the passage?
In most countries, the public sector governance is above the law.

2)

In some countries, governments have managed to step up when required.

3)

The public sector is often blamed for shortcomings in services beyond its scope.

4)

Responsible governments do not rely on performance accountability.

5)

Most governments have lagged behind as they are closed economies.

ite
d

1)

Question: 23
According to the passage, governments have been effective in all the areas except:
crime.

2)

social security.

3)

education.

4)

medical services.

5)

urban infrastructure.

es

Pr

1)

iv
at
e

Li
m

Explanation:
The last paragraph mentions examples of countries where governments have made considerable improvements in critical areas of
development. Thus, they did step up when required. [2] states this. [1] and [3] are not mentioned in the passage. Thus, they can be
negated. [4] is incorrect as the policymakers of these governments did factor in performance accountability. [5] can be negated as the
concept of closed economies is not mentioned. Hence, [2].

es

Question: 24
Which of the following is the author not likely to agree with?

ou

rc

Explanation:
The last paragraph of the passage mentions governments successes in the fields of crime, social security, education and medical
services, but does not mention urban infrastructure. Hence, [5].

The public is becoming increasingly critical of government policies.

2)

The past three decades of financial prosperity have given way to income inequality.

3)

Service delivery can improve in an environment of strong leadership and accountability.

4)

Most public utilities are becoming too unwieldy to be handled alone by the government.

5)

Governments often face the challenge of delivering comprehensively and in a timely manner.

ar

ni

ng

1)

IM

Le

Explanation:
[1] and [2] follow from the second and first paragraph respectively. [3] and [5] follow from the last paragraph. However, there is no
indication of [4] in the passage.
Hence, [4].
Question: 25
Which of the following is not among the reasons the government should feel pressurized to rethink their policies?
1)

Public grievances

2)

Skill mismatch in human resources

3)

Mass migration of labour

4)

Rising cost of capital

IMS Learning Resources Pvt.Ltd.,Mumbai.All copyrights to this material vestswith IMS Learning Resources Pvt.Ltd.
No part of this materials either in part oras a whole shall be copied,printed,electronically reproduced,sold or distributed without the written
consent of IMS Learing Resources Pvt.Ltd.and any such violation would entail initiation of suitable legal proceedings.

Copyright

Login ID:1363169/Student Name:SSN/Overall Score:147

5)

Expanding population of the elderly

Explanation:
According to the passage, governments should feel obliged to change their policies owing to public outrage, skill gaps, cost of capital and
an ageing population. However, mass migration of labour is not mentioned. Hence, [3].

14

2)

46

3)

139

4)

5)

Li
m

1)

ite
d

Question: 26
Find the odd term in the series.
1, 4, 14, 46, 139, 422

Pr

iv
at
e

Explanation:

3)

73

4)

67

5)

79

rc

85

ou

2)

es

61

ng

Explanation:
The series is a series of prime numbers.
61, 67, 71, 73, 79, 83, 89. Hence, [2].

1)

es

Question: 27
61, 67, 71, 73, 79, 83, 85

2)

15

3)

1)

Le

ar

ni

Question: 28
Choose the correct alternative.
220 chocolates were distributed among 80 boys and girls such that each boy gets 4 chocolates and each girl gets 2 chocolates. Find the
difference between the number of boys and the girls.

4)

10

5)

20

IM

25

Explanation:
Let the number of boys be X and the number of girls be Y.
From the given information,
X + Y = 80

IMS Learning Resources Pvt.Ltd.,Mumbai.All copyrights to this material vestswith IMS Learning Resources Pvt.Ltd.
No part of this materials either in part oras a whole shall be copied,printed,electronically reproduced,sold or distributed without the written
consent of IMS Learing Resources Pvt.Ltd.and any such violation would entail initiation of suitable legal proceedings.

Copyright

Login ID:1363169/Student Name:SSN/Overall Score:147

4X + 2Y = 220
Solving the two equations simultaneously, we get
X = 30 and Y = 50
Y X = 20
Hence, [5].

2)

10.00 am

3)

10.30 am

4)

10.30 am

5)

11 am

Li
m

9.30 am

iv
at
e

1)

ite
d

Question: 29
Six friends are going for a picnic. Five friends started their journey in a car at 8 am at a constant speed of 50 km/hour. However, one friend
woke up late and started his journey on his bike at a constant speed of 75 km/hour. At what time should he leave so that he meets his
friends at 2 pm?

ou

rc

es

Pr

Explanation:

es

Question: 30
In a family of 6 people, B is the brother of D, who is Es mother. C is As grandmother who has F as her son-in-law. How is A related to D,
if A has no brother or sister and C has one son and one daughter?
Niece

2)

Nephew

3)

Uncle

4)

Son

5)

Cannot be determined

ar

ni

ng

1)

IM

Le

Explanation:
In the figure, circle represents a female while square represents a male.
From the data, we get,

A is either the nephew or niece of D. Hence, [5].

IMS Learning Resources Pvt.Ltd.,Mumbai.All copyrights to this material vestswith IMS Learning Resources Pvt.Ltd.
No part of this materials either in part oras a whole shall be copied,printed,electronically reproduced,sold or distributed without the written
consent of IMS Learing Resources Pvt.Ltd.and any such violation would entail initiation of suitable legal proceedings.

Copyright

Login ID:1363169/Student Name:SSN/Overall Score:147

Question: 31
Read the information give below and answer the following questions.
Food import consignments currently have to undergo mandatory testing that delays their release from customs. Customs gets blamed for
delays in clearance of consignments even though it may be for want of reports from the concerned agency involved in the process.
What could be the best course of action to ensure rapid clearance of imported food items in India?
India should extract concessions abroad for easier access to Indian foods, as demand for exotic foods in India is growing.

2)

Chocolate and gourmet ingredients rapidly deteriorate into inedible glop.

3)

Customs department should launch a single-window clearance for all food imports except genetically modified foods and
poultry items.

4)

Checks should be conducted only on controversial food products or those with a negative record.

5)

Random checks should be incorporated, besides giving a time frame to laboratory testing companies.

Li
m

ite
d

1)

iv
at
e

Explanation:
[1] does not address the issue of rapid clearance in India. [2] is not a course of action. While single-window clearance is a practical
course of action, it is partial to certain foods. So, it can be negated. [4] can also be negated as it focuses only onproducts with a negative
record. Only [5] is a practical course of action that can ensure rapid clearance of imported food items in India. Hence, [5].

rc

The above passage makes which of the following assumptions?

es

Pr

Question: 32
In the late nineties, the AP govt gave a boost to the Pre-engineered Building (PEB) industry by offering a seven year tax holiday and
encouraging it to compete with the more prominent RCC industries across India. Later Uttaranchal in 2005 placed PEBs in the scheduled
list and made the applicable tax five per cent. This resulted in many PEB sales offices working out of Uttaranchal.

Successes in boosting a new market are few and far between in a country like India.

2)

The PEB market is likely to grow by capitalizing on the Make in India initiative.

3)

Financial incentives can increase the viability of a relatively new industry.

4)

No new business venture will succeed without government support.

5)

Emulating an internationally successful model in a local scenario is time-consuming.

es

ou

1)

ar

ni

ng

Explanation:
The assumption in the passage is that tax incentives since the 1990s have increased the number of PEB sales offices and made it more
viable in the state that gave it an additional tax break. [3] contains this assumption. There is nothing in the passage to substantiate [2]. [1]
and [4] can be eliminated because both are generalizations. There is no indication in the passage for us to assume [5]. Hence, [3].

Setting up high-security hostels specifically for working women

IM

1)

Le

Question: 33
While women are as smart as men and are eager to participate in work, when the job involves relocation, concerns of parents curb the
aspirations of the potential workers. Thoughts of safety, relocation costs and lifestyle differences discourage parents from allowing their
daughters to relocate for work. Eventually, the girls end up letting go of good opportunities or not seeking them.
Which of the following can address the issue of relocation?

2)

Not employing women in decision-making roles

3)

Providing transport to and from work and offering flexible work timings

4)

Giving a preference to women who have taken career breaks

5)

Making the workplace women-friendly

Explanation:

IMS Learning Resources Pvt.Ltd.,Mumbai.All copyrights to this material vestswith IMS Learning Resources Pvt.Ltd.
No part of this materials either in part oras a whole shall be copied,printed,electronically reproduced,sold or distributed without the written
consent of IMS Learing Resources Pvt.Ltd.and any such violation would entail initiation of suitable legal proceedings.

Copyright

Login ID:1363169/Student Name:SSN/Overall Score:147

Only [1] is likely to make parents agree to the idea of relocation. [2] distracts from the main issue of relocation, and is a negative
suggestion. [3] and [5] would be helpful, but do not address the issue of relocation. [4] is not related to the issue of womens safety, or
relocation costs. Hence, [1].

India is one of the largest world markets for online bookings.

2)

Indians are more digitally savvy than many other Asian nations.

3)

Indians are budget conscious travellers and adhere to companies travel policies.

4)

Indian travellers like to spend on luxuries during business trips.

5)

Indian business travellers demand access to technology both in-flight and during stays.

Li
m

1)

ite
d

Question: 34
Travel trends in a number of key Asian markets suggest the highest percentage of Indians preferred online medium for their hotel
bookings. Travel companies reporting a lions share of bookings through mobile applications and hotel chains roll out digital check-in and
check-out processes to woo Indians.
Which of the following follows from the above paragraph?

es

Pr

iv
at
e

Explanation:
[1] can be negated because of the term world, as information is provided only on Asian markets in this paragraph. [2] follows from the
trends that suggest the highest percentage of Indians make their bookings via online portals. [3] can be eliminated as it contradicts the
paragraph. [4] can be negated as there is no information on luxury spending. [5] can also be negated as there is no mention of Indians
demanding access to technology (rather, the implication is that they already have access).
Hence, [2].

ou

rc

Question: 35
Top recruiters in India have been flocking to B-schools much ahead of the campus recruitment stage and using live projects, promises of
funding startup ideas and strategy competitions as inducement to engage with and identify new talent. The inclusion of first year students
marks the growing importance being accorded to tapping this talent pool early on.
What could be the reason for the popularity of the new hiring strategy?
Students can try their luck by being exposed to work cultures of different companies.

2)

Recruiting companies can gauge students abilities by evaluating their strategic thinking skills.

3)

The new system has put paid to the old system of campus visits just before the final semester.

4)

The earlier process was long-drawn-out and involved long procedures of formal interviews.

5)

Hiring through alumni network, website and social media is very cost effective for companies.

ng

es

1)

Le

ar

ni

Explanation:
[1] can be negated as it is from the students point of view, not the companies. [2] presents a valid reason for the popularity of the new
hiring strategy. [3] is not a reason and thus can be negated. [4] can also be eliminated as there is no indication that the new system will
not involve long procedures as well. [5] focuses on off-campus recruitment and can be eliminated. Hence, [2].

IM

Question: 36
Refer to the data below and answer the questions that follow.
A word arrangement machine when given an input line of words rearranges them following a particular rule in each step. The following is
an illustration of input and rearrangement.
Input: false set impression your spirit research
Step 1: set false impression your spirit research
Step 2: set your false impression spirit research
Step 3: set your false spirit impression research
Step 4: set your false spirit research impression

IMS Learning Resources Pvt.Ltd.,Mumbai.All copyrights to this material vestswith IMS Learning Resources Pvt.Ltd.
No part of this materials either in part oras a whole shall be copied,printed,electronically reproduced,sold or distributed without the written
consent of IMS Learing Resources Pvt.Ltd.and any such violation would entail initiation of suitable legal proceedings.

Copyright

Login ID:1363169/Student Name:SSN/Overall Score:147

Step 4 is the final output


If the input is achieve can we successfully everything together, what will be the step 3?
1)

we can together achieve successfully everything

2)

we can achieve together everything successfully

3)

we achieve can together everything successfully

4)

we can achieve together successfully everything

5)

we can achieve everything successfully together

iv
at
e

Li
m

Input - achieve can we successfully everything together


Step 1 we achieve can successfully everything together
Step 2 we can achieve successfully everything together
Step 3 we can achieve together successfully everything
Hence, [4].

ite
d

Explanation:
Logic here is to arrange the words in the increasing order of number of letters from left to right.

Question: 37
What will be the position of everything in the 5th step if input is same as in the previous question?
Third from the left end

2)

Extreme right end

3)

Third from the right end

4)

Second from the left end

5)

There is no such step

ou

rc

es

Pr

1)

ar

ni

ng

es

Explanation:
Logic here is to arrange the words in the increasing order of number of letters from left to right.
Input achieve can we successfully everything together
Step 1 we achieve can successfully everything together
Step 2 we can achieve successfully everything together
Step 3 we can achieve together successfully everything
Step 4 we can achieve together everything successfully
As the words are arranged in increasing order of number of letters, Step 4 is thefinal step.
Therefore, there is no 5th step.
Hence, [5].

1)

Third from the left

2)

Le

Question: 38
If the input is behavior your is awesome simultaneously and similarly what will be the position of the word awesome in the 4th step?

IM

Fourth from the left

3)

Third from the right

4)

Second from the right

5)

Second from the left

Explanation:
Logic here is to arrange the words in the increasing order of number of letters from left to right.

IMS Learning Resources Pvt.Ltd.,Mumbai.All copyrights to this material vestswith IMS Learning Resources Pvt.Ltd.
No part of this materials either in part oras a whole shall be copied,printed,electronically reproduced,sold or distributed without the written
consent of IMS Learing Resources Pvt.Ltd.and any such violation would entail initiation of suitable legal proceedings.

Copyright

Login ID:1363169/Student Name:SSN/Overall Score:147

Input behavior your is awesome simultaneously and similarly


Step 1 is behavior your awesome simultaneously and similarly
Step 2 is and behavior your awesome simultaneously similarly
Step 3 is and your behavior awesome simultaneously similarly
Step 4 is and your awesome behavior simultaneously similarly
The position of the word awesome is fourth from the left.
Hence, [2].

2)

3)

VI

4)

VII

5)

VIII

Li
m

IV

iv
at
e

1)

ite
d

Question: 39
What is the final step for the input in the previous question?

Explanation:
Logic here is to arrange the words in the increasing order of number of letters from left to right.

es

ou

rc

es

Pr

Input behavior your is awesome simultaneously and similarly


Step 1 is behavior your awesome simultaneously and similarly
Step 2 is and behavior your awesome simultaneously similarly
Step 3 is and your behavior awesome simultaneously similarly
Step 4 is and your awesome behavior simultaneously similarly
Step 5 is and your awesome behavior similarly simultaneously
As the words are arranged in increasing order of number of letters, Step 5 is the final step
Hence, [2].

Question: 40
If the input is one should work tirelessly otherwise failure is expected, what will be the final output?
is one work failure should otherwise expected tirelessly

2)

is one work should failure otherwise expected tirelessly

3)

is one work should failure expected otherwise tirelessly

4)

is one work should failure otherwise expected tirelessly

5)

is one work should otherwise failure expected tirelessly

ar

ni

ng

1)

Le

Explanation:
The words are 3, 6, 4, 10, 9, 7, 2 and 8 letters long respectively. When the input is sorted in terms of ascending order of number of letters
in the words, the final output is is one work should failure expected otherwise tirelessly. Hence, [3].

IM

Question: 41
The first two words in each question have a certain relationship with each other. Choose the option that fits in the blank in such
a way that the third and fourth words also have the same relationship to each other.
Sloth : Bears :: _____ : Bees
1)

Cloud

2)

Drift

3)

Herd

IMS Learning Resources Pvt.Ltd.,Mumbai.All copyrights to this material vestswith IMS Learning Resources Pvt.Ltd.
No part of this materials either in part oras a whole shall be copied,printed,electronically reproduced,sold or distributed without the written
consent of IMS Learing Resources Pvt.Ltd.and any such violation would entail initiation of suitable legal proceedings.

Copyright

Login ID:1363169/Student Name:SSN/Overall Score:147

4)

Colony

5)

School

Explanation:
A group of bears is called a sloth of bears. Similarly, a group of bees is collectively called a colony of bees. Hence, [4].

Building

2)

Crockery

3)

Clothes

4)

Food

5)

Painting

Li
m

1)

ite
d

Question: 42
Wicker : Furniture :: Porcelain : _____

iv
at
e

Explanation:
The analogy is of raw material and product. Wicker is used for making furniture. Similarly, porcelain is used for making crockery.
Hence, [2].

Sip

2)

Pat

3)

Brush

4)

Kick

5)

Walk

ou

rc

1)

es

Pr

Question: 43
Morsel : Food :: _____ : Drink

es

Explanation:
We eat a morsel of food, we take a sip of a drink. Hence, [1].

3)

Blessing

4)

Words

5)

Distance

ng

Space

ni

2)

ar

Time

Le

1)

Question: 44
Gustatory : Taste :: Temporal : _____

Explanation:
Gustatory relates to taste while temporal relates to time. Hence, [1].

IM

Question: 45
Perspective : Design :: _____ : Photography
1)

Dirge

2)

Timbre

3)

Montage

4)

Stance

IMS Learning Resources Pvt.Ltd.,Mumbai.All copyrights to this material vestswith IMS Learning Resources Pvt.Ltd.
No part of this materials either in part oras a whole shall be copied,printed,electronically reproduced,sold or distributed without the written
consent of IMS Learing Resources Pvt.Ltd.and any such violation would entail initiation of suitable legal proceedings.

Copyright

Login ID:1363169/Student Name:SSN/Overall Score:147

5)

Pipe

Explanation:
Just as perspective is a term pertaining to design, montage is a term pertaining to photography. Hence, [3].

4:5

2)

3:4

3)

2:3

4)

5:4

5)

4:3

Li
m

1)

ite
d

Question: 46
Choose the correct alternative.
Two different varieties of Rice cost Rs.15 per kg and Rs. 24 per kg. In what ratio should these two quantities be mixed so that we can
make a profit of 25% after selling the mixture for Rs.25 per kg?

ni

ng

es

ou

rc

es

Pr

iv
at
e

Explanation:

2)

11

3)

10

IM

4)

Le

1)

ar

Question: 47
What is the value of k if kX2 12X + 4 = 0 has equal roots?

5)

12

Explanation:

IMS Learning Resources Pvt.Ltd.,Mumbai.All copyrights to this material vestswith IMS Learning Resources Pvt.Ltd.
No part of this materials either in part oras a whole shall be copied,printed,electronically reproduced,sold or distributed without the written
consent of IMS Learing Resources Pvt.Ltd.and any such violation would entail initiation of suitable legal proceedings.

Copyright

Login ID:1363169/Student Name:SSN/Overall Score:147

10 days

2)

12 days

3)

8 days

4)

6 days

5)

9 days

Li
m

1)

ite
d

Question: 48
A can do a piece of work in 15 days. How much time will be required to complete the same work for B who is 50% more efficient than A?

rc

es

Pr

iv
at
e

Explanation:

13 years

3)

18 years

4)

22 years

5)

25 years

es

2)

8 years

ng

1)

ou

Question: 49
Varsha is 46 years old while her son Raghu is 12 years old. After how many years will she be twice as old as her son?

Le

ar

ni

Explanation:

IM

Question: 50
If the length of the diagonal of a rectangle is 20 cm with one side as 12 cm, then find the area of the rectangle.
1)
172 cm2
2)

164 cm2

3)

216 cm2

4)

248 cm2

5)

192 cm2

IMS Learning Resources Pvt.Ltd.,Mumbai.All copyrights to this material vestswith IMS Learning Resources Pvt.Ltd.
No part of this materials either in part oras a whole shall be copied,printed,electronically reproduced,sold or distributed without the written
consent of IMS Learing Resources Pvt.Ltd.and any such violation would entail initiation of suitable legal proceedings.

Copyright

Login ID:1363169/Student Name:SSN/Overall Score:147

Li
m

ite
d

Explanation:

DABCE

3)

ADCEB

4)

BECAD

5)

BADEC

rc

2)

ou

BEDAC

es

1)

es

Pr

iv
at
e

Question: 51
Choose from among the five options, the option that identifies a logical sequence that should be followed by the figures.
.

ng

Explanation:
The larger figure rotates 90 clockwise in every alternate step, while the small arrow moves from the left to the right of the main figure.
Hence, [3].

IM

Le

ar

ni

Question: 52
Each question contains a sequence beginning with a diagram labelled [X]. Of the 5 other diagrams labelled [1] to [5], two adjacent
diagrams may have been interchanged. For your answers, identify the two adjacent diagrams that need to be interchanged to form a
logical sequence, and mark the first diagram as your answer. In case no interchange is required, then choose [5].

1)

2)

3)

4)

IMS Learning Resources Pvt.Ltd.,Mumbai.All copyrights to this material vestswith IMS Learning Resources Pvt.Ltd.
No part of this materials either in part oras a whole shall be copied,printed,electronically reproduced,sold or distributed without the written
consent of IMS Learing Resources Pvt.Ltd.and any such violation would entail initiation of suitable legal proceedings.

Copyright

Login ID:1363169/Student Name:SSN/Overall Score:147

5)

Explanation:
Two steps are followed alternatively. In the first step, the top left symbol is water imaged and then rotated 45 anticlockwise. In the next
step, the top left symbol is mirror imaged and rotated 45 anticlockwise and so on. The bottom left symbol is mirrored and rotated 90
clockwise in step one and is water imaged and rotated 90 clockwise in next step and so on. The central symbol is rotated 90 clockwise
in every step. Thus, [3] and [4] need to be interchanged.
Hence, [3].

3)

4)

5)

Pr

2)

es

rc

1)

iv
at
e

Li
m

ite
d

Question: 53
Each question contains a sequence beginning with a diagram labelled [X]. Of the 5 other diagrams labelled [1] to [5], two adjacent
diagrams may have been interchanged. For your answers, identify the two adjacent diagrams that need to be interchanged to form a
logical sequence, and mark the first diagram as your answer. In case no interchange is required, then choose [5].

es

ou

Explanation:
Two steps are followed here alternately. In one step the entire block is inverted horizontally and then rotated 90 anticlockwise and in the
successive step each element is again inverted vertically and moved two places ahead. Thus, to form a correct logical series, [1] and [2]
need to be interchanged.Hence, [1].

Le

ar

ni

ng

Question: 54
Each question contains a sequence beginning with a diagram labelled [X]. Of the 5 other diagrams labelled [1] to [5], two adjacent
diagrams may have been interchanged. For your answers, identify the two adjacent diagrams that need to be interchanged to form a
logical sequence, and mark the first diagram as your answer. In case no interchange is required, then choose [5].

IM

2)

1)

3)

4)

5)

Explanation:
Two steps are followed here alternately. In the first step the top and bottom symbols interchange positions, also the right symbol moves to

IMS Learning Resources Pvt.Ltd.,Mumbai.All copyrights to this material vestswith IMS Learning Resources Pvt.Ltd.
No part of this materials either in part oras a whole shall be copied,printed,electronically reproduced,sold or distributed without the written
consent of IMS Learing Resources Pvt.Ltd.and any such violation would entail initiation of suitable legal proceedings.

Copyright

Login ID:1363169/Student Name:SSN/Overall Score:147

left and a new symbol is added to the right. In the second step all the symbols move one position ahead in anticlockwise direction. Thus,
[3] and [4] need to be interchanged to form a correct sequence. Hence, [3].

2)

3)

4)

5)

iv
at
e

Pr

1)

Li
m

ite
d

Question: 55
Each question contains a sequence beginning with a diagram labelled [X]. Of the 5 other diagrams labelled [1] to [5], two adjacent
diagrams may have been interchanged. For your answers, identify the two adjacent diagrams that need to be interchanged to form a
logical sequence, and mark the first diagram as your answer. In case no interchange is required, then choose [5].
.

ou

rc

es

Explanation:

IM

Le

ar

ni

ng

es

Question: 56
Refer to the data below and answer the questions that follow.
Following line graph shows the total sales (in Rs. Lakhs) of three products of a company, named Product A, Product B and Product C over
6 years (2009 to 2014). These are the only products sold by the company. Study the graph and answer the questions that follow.

Which of the following years showed highest total sales?


1)

2011

IMS Learning Resources Pvt.Ltd.,Mumbai.All copyrights to this material vestswith IMS Learning Resources Pvt.Ltd.
No part of this materials either in part oras a whole shall be copied,printed,electronically reproduced,sold or distributed without the written
consent of IMS Learing Resources Pvt.Ltd.and any such violation would entail initiation of suitable legal proceedings.

Copyright

Login ID:1363169/Student Name:SSN/Overall Score:147

2)

2014

3)

2010

4)

2009

5)

2013

Explanation:
We can generate the following approximate table from the graph:

iv
at
e

Li
m

ite
d

Now all the questions can be answered.

Pr

Hence, [5].

2010

3)

2011

4)

2012

5)

2014

rc

2)

ou

2009

es

1)

es

Question: 57
Which year showed the maximum difference between the sales of Product A and Product B?

ng

Explanation:
We can generate the following approximate table from the graph:

IM

Le

ar

ni

Now all the questions can be answered.

Hence, [1].
Question: 58
What is the approximate yearly average value of sales of Product C over the entire period (in Rs. Lakhs)?
1)

375

IMS Learning Resources Pvt.Ltd.,Mumbai.All copyrights to this material vestswith IMS Learning Resources Pvt.Ltd.
No part of this materials either in part oras a whole shall be copied,printed,electronically reproduced,sold or distributed without the written
consent of IMS Learing Resources Pvt.Ltd.and any such violation would entail initiation of suitable legal proceedings.

Copyright

Login ID:1363169/Student Name:SSN/Overall Score:147

2)

450

3)

525

4)

575

5)

350

Explanation:
We can generate the following approximate table from the graph:

es

Pr

iv
at
e

Li
m

ite
d

Now all the questions can be answered.

25%

3)

55%

4)

30%

5)

45%

es

2)

40%

ng

1)

ou

rc

Question: 59
What was the maximum percent decrease in sales of any product in any year with respect to the sales previous year?

ni

Explanation:
We can generate the following approximate table from the graph:

IM

Le

ar

Now all the questions can be answered.

IMS Learning Resources Pvt.Ltd.,Mumbai.All copyrights to this material vestswith IMS Learning Resources Pvt.Ltd.
No part of this materials either in part oras a whole shall be copied,printed,electronically reproduced,sold or distributed without the written
consent of IMS Learing Resources Pvt.Ltd.and any such violation would entail initiation of suitable legal proceedings.

Copyright

Login ID:1363169/Student Name:SSN/Overall Score:147

2)

15%

3)

30%

4)

50%

5)

60%

Li
m

20%

iv
at
e

1)

ite
d

Question: 60
By what percent is the total sale of product C over the years greater than the total sale of product A over the years?

Pr

Explanation:
We can generate the following approximate table from the graph:

ar

ni

ng

es

ou

rc

es

Now all the questions can be answered.

IM

Le

Question: 61
Read the sentences given below carefully. Parts of the sentences have been underlined as [1], [2], [3] and [4]. If there is an error in the
sentence as a result of any of these parts, mark the relevant part. If there are no errors then mark [5].

1)

2)

3)

IMS Learning Resources Pvt.Ltd.,Mumbai.All copyrights to this material vestswith IMS Learning Resources Pvt.Ltd.
No part of this materials either in part oras a whole shall be copied,printed,electronically reproduced,sold or distributed without the written
consent of IMS Learing Resources Pvt.Ltd.and any such violation would entail initiation of suitable legal proceedings.

Copyright

Login ID:1363169/Student Name:SSN/Overall Score:147

4)

5)

Explanation:
The correct usage is another spell of rain. Hence, [4].

2)

3)

4)

5)

Pr

es

1)

iv
at
e

Li
m

ite
d

Question: 62

rc

Explanation:
In the context of the sentence, the correct word is deflect, not detect. Deflect is always followed by from. Hence, [2].

3)

4)

5)

ni

2)

ar

Le

1)

ng

es

ou

Question: 63

IM

Explanation:
Something is second onlyto something. So, livestock and seed are second only to land and water, not over. Hence, [2].
Question: 64

IMS Learning Resources Pvt.Ltd.,Mumbai.All copyrights to this material vestswith IMS Learning Resources Pvt.Ltd.
No part of this materials either in part oras a whole shall be copied,printed,electronically reproduced,sold or distributed without the written
consent of IMS Learing Resources Pvt.Ltd.and any such violation would entail initiation of suitable legal proceedings.

Copyright

2)

3)

4)

5)

Li
m

1)

ite
d

Login ID:1363169/Student Name:SSN/Overall Score:147

Explanation:
In the context of the sentence, the correct word is blurred. Hence, [2].

3)

4)

5)

rc

ou

2)

Explanation:
There is no error in the sentence. Hence, [5].

es

1)

es

Pr

iv
at
e

Question: 65

IM

Le

ar

ni

ng

Question: 66
Each question is followed by two statements I and II.
Mark [1], if the question can be answered by using statement I alone.
Mark [2], if the question can be answered by using statement II alone.
Mark [3], if the question can be answered by using either of the statements alone.
Mark [4], if the question can be answered by using both the statements together, but cannot be answered by using either of the
statements alone.
Mark [5], if more data is required to answer the question.
A 3-digit number is greater than 300. What is its value?
I. The units digit is greater than the tens digit.
II. The product of the digits is 9.
1)

2)

3)

4)

5)

IMS Learning Resources Pvt.Ltd.,Mumbai.All copyrights to this material vestswith IMS Learning Resources Pvt.Ltd.
No part of this materials either in part oras a whole shall be copied,printed,electronically reproduced,sold or distributed without the written
consent of IMS Learing Resources Pvt.Ltd.and any such violation would entail initiation of suitable legal proceedings.

Copyright

Login ID:1363169/Student Name:SSN/Overall Score:147

Explanation:
From statement I, we can get multiple possibilities. Therefore, this statement is not sufficient to answer the question.
From statement II, we get three possibilities 911, 331 and 313. Therefore, this statement is also not sufficient to answer the question.
Combining the two statements, we get the answer as 313.
Hence, [4].

2)

3)

4)

5)

Li
m

iv
at
e

1)

ite
d

Question: 67
Is the 6-digit number 3x8y4z divisible by 9?
I. x + y + z = 12
II. Product of x, y and z is 315

3)

4)

5)

rc

2)

ng

ni

1)

es

ou

Question: 68
Is the area of circle A larger than that of circle B?
I. The circumference of circle A is greater than that of circle B.
II. The largest chord of circle A is greater than that of circle B.

es

Pr

Explanation:
Consider statement I. Sum of all the digits of the number = 3 + 8 + 4 + 12 = 27. Since 27 is divisible by 9, therefore the number is divisible
by 9. Therefore, this statement is sufficient to answer the question.
Consider statement II. Since xyz = 315 = 5 7 9, the values of x, y and z will be 5, 7 and 9 (in no particular order). Therefore, the sum of
all the digits will be 3 + 8 + 4 + 5 + 7 + 9 = 36. Since 36 is divisible by 9, therefore the number is divisible by 9. Therefore, this statement is
sufficient to answer the question.
Hence, [3].

IM

Le

ar

Explanation:
Greater the circumference, greater would be the radius, which means greater the area. Therefore, statement I is sufficient to answer the
question.
Greater the largest chord (diameter), greater would be the radius, which means greater the area. Therefore, statement II is sufficient to
answer the question.
Hence, [3].
Question: 69
What is the radius of the cylindrical vessel?
I. When 1 litre of water is added into the vessel, the level of water increases by one-tenth.
II. When 1 litre of water is removed from the vessel, the level of water drops by 1 cm.
1)

2)

IMS Learning Resources Pvt.Ltd.,Mumbai.All copyrights to this material vestswith IMS Learning Resources Pvt.Ltd.
No part of this materials either in part oras a whole shall be copied,printed,electronically reproduced,sold or distributed without the written
consent of IMS Learing Resources Pvt.Ltd.and any such violation would entail initiation of suitable legal proceedings.

Copyright

Login ID:1363169/Student Name:SSN/Overall Score:147

3)

4)

5)

ite
d

Explanation:
Consider statement I. For the increase in water level, we know the volume, but we dont know the exact value of the height. Thus, the
radius cannot be determined. Therefore, this statement is not sufficient to answer the question.
Consider statement II. For the decrease in water level, we know the volume and the value of the height of water level drop. Thus, using
the formula of volume of a cylinder, the radius can be determined. Therefore, this statement is sufficient to answer the question. Hence,
[2].

2)

3)

4)

5)

Pr

es

1)

iv
at
e

Li
m

Question: 70

es

ou

rc

Explanation:

ng

Question: 71
Arrange the given words in a meaningful sequence:

evaporation, precipitation, condensation, infiltration, runoff

2)

condensation, evaporation, precipitation, infiltration, runoff

3)

precipitation, condensation, runoff, infiltration, evaporation

4)

precipitation, condensation, runoff, evaporation, infiltration

5)

evaporation, condensation, precipitation, runoff, infiltration

Le

ar

ni

1)

IM

Explanation:
The correct sequence in a water cycle is evaporation, condensation, precipitation, runoff and infiltration. Hence, [5].
Question: 72
1)

takeoff, cruise, descent, landing, taxi

2)

descent, takeoff, cruise, taxi, landing

3)

cruise, takeoff, taxi, descent , landing

4)

taxi, descent, takeoff, landing, cruise

IMS Learning Resources Pvt.Ltd.,Mumbai.All copyrights to this material vestswith IMS Learning Resources Pvt.Ltd.
No part of this materials either in part oras a whole shall be copied,printed,electronically reproduced,sold or distributed without the written
consent of IMS Learing Resources Pvt.Ltd.and any such violation would entail initiation of suitable legal proceedings.

Copyright

Login ID:1363169/Student Name:SSN/Overall Score:147

5)

taxi, takeoff, cruise, descent, landing

Explanation:
[5] correctly presents the sequence in which a flight moves from start to landing. Hence, [5].
Question: 73
Choose the option which correctly converts the given sentence into indirect speech.
No country shall be allowed to throw a region into chaos for selfish gain, the President said.
The President said no country shall be allowed to throw a region into chaos for selfish gain.

2)

The President explained that no country be ever allowed to throw a region into chaos for selfish gain.

3)

The President said that no country is to be allowed to throw a region into chaos for selfish gain.

4)

The President felt that no country should be allowed to throw a region into chaos for selfish gain.

5)

The President asserted that no country should be allowed to throw a region into chaos for selfish gain.

Li
m

ite
d

1)

Pr

Question: 74
A fan said about his favourite musician I cant imagine him with a frown.

iv
at
e

Explanation:
In indirect speech, the present modal is changed into past modal. In the indirect speech, shall should be changed into should. [1] can be
negated on this basis. Since [2] and [3] do not have the modal verb, they can be negated. Since the statement is a strong one, asserted
makes more sense as the reporting verb than felt. So, [4] can be ruled out, and only [5] converts the direct speech into indirect speech
correctly. Hence, [5].

A fan said he couldnt imagine his favourite musicians frown.

2)

A fan said about his favourite musician that he couldnt imagine him with a frown.

3)

A fan said his favourite musician how he couldnt imagine him with a frown.

4)

A fan said about his favourite musician that he cant imagine him with a frown.

5)

A fan said about his favourite musician that couldnt ever imagine him with a frown.

es

ou

rc

es

1)

ng

Explanation:
[1] distorts the meaning of the statement. Thus, it can be negated. According to the rules of changing direct speech to indirect speech, the
present tense should be changed to the past tense. [4] does not follow this rule and can be negated. [3] wrongly uses how and can be
negated as it changes the meaning of the sentence. [5] can be negated as it introduces an adverb ever. Only [2] follows the rules of
conversion correctly. Hence, [2].

The World War II veteran had told the audience that he felt called into serving others.

2)

The World War II veteran urged the audience to be called into serving others.

3)

The World War II veteran told the audience that he felt called into serving others.

4)

Le

1)

ar

ni

Question: 75
The World War II veteran told the audience, I feel called into serving others.

IM

The World War II veteran told the audience that he feels called into serving others.

5)

The World War II veteran told the audience to feel called into serving others.

Explanation:
According to the rules for changing direct into indirect speech, the present tense in the direct becomes past tense in the indirect speech.
[4] can be negated as it does not follow this rule. [2] and [5] distort the meaning of the sentence. [1] can be ruled out as it wrongly uses the
past perfect had told. Hence, [3].
Question: 76

IMS Learning Resources Pvt.Ltd.,Mumbai.All copyrights to this material vestswith IMS Learning Resources Pvt.Ltd.
No part of this materials either in part oras a whole shall be copied,printed,electronically reproduced,sold or distributed without the written
consent of IMS Learing Resources Pvt.Ltd.and any such violation would entail initiation of suitable legal proceedings.

Copyright

Login ID:1363169/Student Name:SSN/Overall Score:147

It is against the law for anyone to repair demolished structures, the engineer said to the Municipal Commissioner.
1)

The engineer told the Municipal Commissioner that it was against the law for anyone to repair demolished structures.

2)

The engineer told the Municipal Commissioner that it were against the law for anyone to repair demolished structures.

3)

The engineer informed the Municipal Commissioner that the law is against anyone to repair demolished structures.

4)

The engineer said to the Municipal Commissioner that it was not the law to repair demolished structures.

5)

The engineer told the Municipal Commissioner to go against the law to repair demolished structures.

ite
d

Explanation:
Only [1] correctly converts the given sentence into indirect speech. [2] is incorrect as it wrongly uses were instead of was. [3] and [4]
distort the meaning of the sentence. [5] completely changes the meaning of the sentence. Thus, all except [1] can be negated. Hence, [1].

Present perfect tense

2)

Present perfect continuous tense

3)

Present continuous tense

4)

Past perfect tense

5)

Past perfect continuous tense

Pr

1)

iv
at
e

Li
m

Question: 77
Choose the correct alternative.
Identify the tense in the following sentence: In the past two years, the University has been trying to take the process online.

ou

rc

es

Explanation:
The given sentence is in the present perfect continuous tense in the form: have been + present participle of the main verb (base + ing). It
denotes that the action was started but did not finish in that period of time and is still going on.
Hence, [2].

2)

Verb

3)

Adverb

4)

Adjective

5)

Participle

ng

Noun

ni

1)

es

Question: 78
Awhile is a/an

Le

ar

Explanation:
The word awhile, spelled as a single word, is an adverb. Hence, [3].
Question: 79
Identify the underlined phrase: We strolled through the beautiful countryside.
Noun phrase

2)

1)

3)

Prepositional phrase

4)

Participial phrase

5)

Verb phrase

IM

Adjective phrase

Explanation:
The phrase through the beautiful countryside is a prepositional phrase. Hence, [3].

IMS Learning Resources Pvt.Ltd.,Mumbai.All copyrights to this material vestswith IMS Learning Resources Pvt.Ltd.
No part of this materials either in part oras a whole shall be copied,printed,electronically reproduced,sold or distributed without the written
consent of IMS Learing Resources Pvt.Ltd.and any such violation would entail initiation of suitable legal proceedings.

Copyright

Login ID:1363169/Student Name:SSN/Overall Score:147

Question: 80
Identify the underlined word in the sentence: Feeling hurt, the man plodded his way down the road.
1)

Perfect participle

2)

Past participle

3)

Infinitive phrase

4)

Present participle

5)

None of the above

ite
d

Explanation:
In the sentence, the word feeling is a present participle. Hence, [4].

Li
m

Question: 81
Refer to the data below and answer the questions that follow.

rc

es

Pr

iv
at
e

Eight friendsA, B, C, D, E, F, G and Hare sitting around a circular table such that four of them are facing the centre of the table and
the remaining four are facing away from the centre of the table. The following information is also known.
Exactly two people are directly facing each other and exactly two people who sit in diametrically opposite positions are facing opposite
directions and not facing each other.
H is sitting two places to the left of C.
D is sitting three places to the right of F.
G and H are facing each other.
E and B are both facing the centre of the table and the only person (who is not G) sitting between them is also facing the centre of the
table.
A is four places to the left of B.
Who is sitting diametrically opposite C?
D

2)

3)

4)

5)

Cannot be determined

ng

es

ou

1)

Le

ar

ni

Explanation:
From the first condition, we can infer that two people are facing each other (G and H), two people are facing away from each other, and
the remaining four people consist of two pairs of persons looking in the same direction (that is, one person out of a pair is facing the centre
of the table and the other is facing away from the centre of the table such that both of them are facing the same direction)

IM

Using conditions 4 and 5, we know that G, H, E and B are the four persons facing the centre of the table. Their arrangement can have two
possibilities as shown below.

IMS Learning Resources Pvt.Ltd.,Mumbai.All copyrights to this material vestswith IMS Learning Resources Pvt.Ltd.
No part of this materials either in part oras a whole shall be copied,printed,electronically reproduced,sold or distributed without the written
consent of IMS Learing Resources Pvt.Ltd.and any such violation would entail initiation of suitable legal proceedings.

Copyright

ite
d

Login ID:1363169/Student Name:SSN/Overall Score:147

es

Pr

iv
at
e

Adding conditions 2 and 6 to the above possibilities, we get the following two possibilities.

Li
m

OR

rc

OR

Le

ar

ni

ng

es

ou

But the first possibility contradicts condition 3. Hence, we get the final arrangement as follows.

IM

Using this, we can answer all the questions.

Hence, [1]

Question: 82
Who is sitting two places to the right of H?
1)

IMS Learning Resources Pvt.Ltd.,Mumbai.All copyrights to this material vestswith IMS Learning Resources Pvt.Ltd.
No part of this materials either in part oras a whole shall be copied,printed,electronically reproduced,sold or distributed without the written
consent of IMS Learing Resources Pvt.Ltd.and any such violation would entail initiation of suitable legal proceedings.

Copyright

Login ID:1363169/Student Name:SSN/Overall Score:147

2)

3)

4)

5)

Cannot be determined

Li
m

ite
d

Explanation:
From the first condition, we can infer that two people are facing each other (G and H), two people are facing away from each other, and
the remaining four people consist of two pairs of persons looking in the same direction (that is, one person out of a pair is facing the centre
of the table and the other is facing away from the centre of the table such that both of them are facing the same direction)

rc

es

Pr

iv
at
e

Using conditions 4 and 5, we know that G, H, E and B are the four persons facing the centre of the table. Their arrangement can have two
possibilities as shown below.

ou

OR

OR

Le

ar

ni

ng

es

Adding conditions 2 and 6 to the above possibilities, we get the following two possibilities.

IM

But the first possibility contradicts condition 3. Hence, we get the final arrangement as follows.

IMS Learning Resources Pvt.Ltd.,Mumbai.All copyrights to this material vestswith IMS Learning Resources Pvt.Ltd.
No part of this materials either in part oras a whole shall be copied,printed,electronically reproduced,sold or distributed without the written
consent of IMS Learing Resources Pvt.Ltd.and any such violation would entail initiation of suitable legal proceedings.

Copyright

Li
m

ite
d

Login ID:1363169/Student Name:SSN/Overall Score:147

iv
at
e

Using this, we can answer all the questions.

Hence, [4].

Pr

Question: 83
Who is sitting to the immediate left of B?
H

2)

3)

4)

5)

Cannot be determined

ou

rc

es

1)

ng

es

Explanation:
From the first condition, we can infer that two people are facing each other (G and H), two people are facing away from each other, and
the remaining four people consist of two pairs of persons looking in the same direction (that is, one person out of a pair is facing the centre
of the table and the other is facing away from the centre of the table such that both of them are facing the same direction)

IM

Le

ar

ni

Using conditions 4 and 5, we know that G, H, E and B are the four persons facing the centre of the table. Their arrangement can have two
possibilities as shown below.

OR

IMS Learning Resources Pvt.Ltd.,Mumbai.All copyrights to this material vestswith IMS Learning Resources Pvt.Ltd.
No part of this materials either in part oras a whole shall be copied,printed,electronically reproduced,sold or distributed without the written
consent of IMS Learing Resources Pvt.Ltd.and any such violation would entail initiation of suitable legal proceedings.

Copyright

Login ID:1363169/Student Name:SSN/Overall Score:147

ite
d

Adding conditions 2 and 6 to the above possibilities, we get the following two possibilities.

ou

rc

es

Pr

iv
at
e

But the first possibility contradicts condition 3. Hence, we get the final arrangement as follows.

Li
m

OR

es

Using this, we can answer all the questions.

ng

Hence, [2].

2)

3)

4)

5)

Le

ar

1)

ni

Question: 84
How many people sit in between E and G (i.e., excluding both and when counting is done from the person to the immediate right of E)?

IM

Cannot be determined

Explanation:
From the first condition, we can infer that two people are facing each other (G and H), two people are facing away from each other, and
the remaining four people consist of two pairs of persons looking in the same direction (that is, one person out of a pair is facing the centre
of the table and the other is facing away from the centre of the table such that both of them are facing the same direction)

IMS Learning Resources Pvt.Ltd.,Mumbai.All copyrights to this material vestswith IMS Learning Resources Pvt.Ltd.
No part of this materials either in part oras a whole shall be copied,printed,electronically reproduced,sold or distributed without the written
consent of IMS Learing Resources Pvt.Ltd.and any such violation would entail initiation of suitable legal proceedings.

Copyright

Login ID:1363169/Student Name:SSN/Overall Score:147

OR

ou

rc

es

Pr

iv
at
e

Adding conditions 2 and 6 to the above possibilities, we get the following two possibilities.

Li
m

ite
d

Using conditions 4 and 5, we know that G, H, E and B are the four persons facing the centre of the table. Their arrangement can have two
possibilities as shown below.

es

OR

IM

Le

ar

ni

ng

But the first possibility contradicts condition 3. Hence, we get the final arrangement as follows.

Using this, we can answer all the questions.

Hence, [2].
Question: 85

IMS Learning Resources Pvt.Ltd.,Mumbai.All copyrights to this material vestswith IMS Learning Resources Pvt.Ltd.
No part of this materials either in part oras a whole shall be copied,printed,electronically reproduced,sold or distributed without the written
consent of IMS Learing Resources Pvt.Ltd.and any such violation would entail initiation of suitable legal proceedings.

Copyright

Login ID:1363169/Student Name:SSN/Overall Score:147

Which among the following combinations represents the people who are facing away from the centre of the table?
1)

FAEC

2)

BAEC

3)

FADC

4)

DBAF

5)

DACG

Li
m

ite
d

Explanation:
From the first condition, we can infer that two people are facing each other (G and H), two people are facing away from each other, and
the remaining four people consist oftwo pairs of persons looking in the same direction (that is, one person out of a pair is facing the centre
of the table and the other is facing away from the centre of the table such that both of them are facing the same direction)

ou

rc

es

Pr

iv
at
e

Using conditions 4 and 5, we know that G, H, E and B are the four persons facing the centre of the table. Their arrangement can have two
possibilities as shown below.

es

OR

IM

Le

ar

ni

ng

Adding conditions 2 and 6 to the above possibilities, we get the following two possibilities.

OR

But the first possibility contradicts condition 3. Hence, we get the final arrangement as follows.

IMS Learning Resources Pvt.Ltd.,Mumbai.All copyrights to this material vestswith IMS Learning Resources Pvt.Ltd.
No part of this materials either in part oras a whole shall be copied,printed,electronically reproduced,sold or distributed without the written
consent of IMS Learing Resources Pvt.Ltd.and any such violation would entail initiation of suitable legal proceedings.

Copyright

Li
m

ite
d

Login ID:1363169/Student Name:SSN/Overall Score:147

iv
at
e

Using this, we can answer all the questions.

Hence, [3].

2)

D sits to the immediate left of A.

3)

C sits to the immediate right of B.

4)

B and A are looking in the same direction.

5)

C and D are looking in the same direction.

rc

F and E are directly facing each other.

ou

1)

es

Pr

Question: 86
Which of the following statements is correct?

ng

es

Explanation:
From the first condition, we can infer that two people are facing each other (G and H), two people are facing away from each other, and
the remaining four people consist of two pairs of persons looking in the same direction (that is, one person out of a pair is facing the centre
of the table and the other is facing away from the centre of the table such that both of them are facing the same direction)

IM

Le

ar

ni

Using conditions 4 and 5, we know that G, H, E and B are the four persons facing the centre of the table. Their arrangement can have two
possibilities as shown below.

OR

IMS Learning Resources Pvt.Ltd.,Mumbai.All copyrights to this material vestswith IMS Learning Resources Pvt.Ltd.
No part of this materials either in part oras a whole shall be copied,printed,electronically reproduced,sold or distributed without the written
consent of IMS Learing Resources Pvt.Ltd.and any such violation would entail initiation of suitable legal proceedings.

Copyright

Login ID:1363169/Student Name:SSN/Overall Score:147

ite
d

Adding conditions 2 and 6 to the above possibilities, we get the following two possibilities.

ou

rc

es

Pr

iv
at
e

But the first possibility contradicts condition 3. Hence, we get the final arrangement as follows.

Li
m

OR

es

Using this, we can answer all the questions.

ng

Hence, [4].

CG

2)

EA

3)

HD

4)

BE

5)

Le

ar

1)

ni

Question: 87
Which of the following pairs of people sit next to each other?

IM

FG

Explanation:
From the first condition, we can infer that two people are facing each other (G and H), two people are facing away from each other, and
the remaining four people consist of two pairs of persons looking in the same direction (that is, one person out of a pair is facing the centre
of the table and the other is facing away from the centre of the table such that both of them are facing the same direction)

IMS Learning Resources Pvt.Ltd.,Mumbai.All copyrights to this material vestswith IMS Learning Resources Pvt.Ltd.
No part of this materials either in part oras a whole shall be copied,printed,electronically reproduced,sold or distributed without the written
consent of IMS Learing Resources Pvt.Ltd.and any such violation would entail initiation of suitable legal proceedings.

Copyright

Login ID:1363169/Student Name:SSN/Overall Score:147

OR

ou

rc

es

Pr

iv
at
e

Adding conditions 2 and 6 to the above possibilities, we get the following two possibilities.

Li
m

ite
d

Using conditions 4 and 5, we know that G, H, E and B are the four persons facing the centre of the table. Their arrangement can have two
possibilities as shown below.

es

OR

IM

Le

ar

ni

ng

But the first possibility contradicts condition 3. Hence, we get the final arrangement as follows.

Using this, we can answer all the questions.

Hence, [5].
Question: 88

IMS Learning Resources Pvt.Ltd.,Mumbai.All copyrights to this material vestswith IMS Learning Resources Pvt.Ltd.
No part of this materials either in part oras a whole shall be copied,printed,electronically reproduced,sold or distributed without the written
consent of IMS Learing Resources Pvt.Ltd.and any such violation would entail initiation of suitable legal proceedings.

Copyright

Login ID:1363169/Student Name:SSN/Overall Score:147

Choose the correct alternative.


In a regular hexagon, find the measure of interior angle.
1)

108 degrees

2)

120 degrees

3)

135 degrees

4)

145 degrees

5)

144 degrees

Pr

iv
at
e

Li
m

ite
d

Explanation:

rc

es

Question: 89

es

ou

1)

2)

ng

3)
All are equal

5)

cannot be detemined.

ar

ni

4)

IM

Le

Explanation:

IMS Learning Resources Pvt.Ltd.,Mumbai.All copyrights to this material vestswith IMS Learning Resources Pvt.Ltd.
No part of this materials either in part oras a whole shall be copied,printed,electronically reproduced,sold or distributed without the written
consent of IMS Learing Resources Pvt.Ltd.and any such violation would entail initiation of suitable legal proceedings.

Copyright

ou

rc

es

Pr

iv
at
e

Li
m

ite
d

Login ID:1363169/Student Name:SSN/Overall Score:147

40

3)

48

4)

49

5)

51

2)

ng

56

ni

1)

es

Question: 90
1 + 4 3 9 of (6 2) = ?

IM

Le

ar

Explanation:

Question: 91

IMS Learning Resources Pvt.Ltd.,Mumbai.All copyrights to this material vestswith IMS Learning Resources Pvt.Ltd.
No part of this materials either in part oras a whole shall be copied,printed,electronically reproduced,sold or distributed without the written
consent of IMS Learing Resources Pvt.Ltd.and any such violation would entail initiation of suitable legal proceedings.

Copyright

Login ID:1363169/Student Name:SSN/Overall Score:147

Fill in the blanks with the most appropriate option.


Indias advantage over China lies in its democratic credentials and years of trade and cultural ______.
1)

affixes

2)

berths

3)

joints

4)

threads

5)

ties

Li
m

ite
d

Explanation:
Since the sentence talks about culture and trade of a country, the most appropriate word is ties. All others do not fit in the context of the
sentence. Hence, [5].

vague

2)

fixed

3)

pervasive

4)

invasive

5)

wily

Pr

1)

iv
at
e

Question: 92
The misuse of the law as an instrument of harassment is ______ in India.

rc

es

Explanation:
Only pervasive is correct as misuse of a law cannot be vague or fixed. Invasive and wily can also be negated for the same reason.
Hence, [3].

alter

3)

dissuade

4)

persuade

5)

declare

2)

ng

encourage

ni

1)

es

ou

Question: 93
Officials should educate the public about masks that filter out at least some particulate matter, and even subsidize them to ______
adoption.

Le

ar

Explanation:
Since officials should educate the public, subsidizing will encourage adoption. Persuade is followed by to. So it can be negated. Since
dissuade and alter contradict the first part of the sentence, they can be eliminated. Hence, [1].

IM

Question: 94
Several governments that have so far persisted with crippling austerity measures to cut back on welfare spending should reconsider their
______.
1)

aspect

2)

front

3)

stunt

4)

stance

5)

temper

Explanation:

IMS Learning Resources Pvt.Ltd.,Mumbai.All copyrights to this material vestswith IMS Learning Resources Pvt.Ltd.
No part of this materials either in part oras a whole shall be copied,printed,electronically reproduced,sold or distributed without the written
consent of IMS Learing Resources Pvt.Ltd.and any such violation would entail initiation of suitable legal proceedings.

Copyright

Login ID:1363169/Student Name:SSN/Overall Score:147

According to the sentence, persisting with crippling austerity measures is a stance i.e. position or attitude which governments
should reconsider. All the other words do not fit in the context. Hence, [4].

admiration

2)

realization

3)

consummation

4)

negotiation

5)

resolution

Explanation:
Only realization, which means awarenes or recognition, fits the blank correctly. Hence, [2].

iv
at
e

Question: 96

Li
m

1)

ite
d

Question: 95
One of the reasons the military agreed to a transition to a more democratic set-up was the ______ that it could not rule the country with an
iron fist forever.

es

Pr

A ! B means A is neither smaller than nor equal to B


A @ B means A is neither greater than nor equal to B
A # B means A is neither greater than nor smaller than B
A $ B means A is not smaller than B
A % B means A is not greater than B

3)

4)

5)

ng

2)

ni

ar

1)

es

ou

rc

In each question, three statements showing relationships have been given and followed by two conclusions.
Mark [a], if only conclusion I is definitely true.
Mark [b], if only conclusion II is definitely true.
Mark [c], if either conclusion I or II is definitely true.
Mark [d], if both conclusions are definitely true.
Mark [e], if no conclusion is definitely true.
P ! Q, Q # R, R @ S
I. P ! S II. P % S

IM

Le

Explanation:

Question: 97
X @ Y, Y $ Z, Z ! W
I. X # W II. X # Z

IMS Learning Resources Pvt.Ltd.,Mumbai.All copyrights to this material vestswith IMS Learning Resources Pvt.Ltd.
No part of this materials either in part oras a whole shall be copied,printed,electronically reproduced,sold or distributed without the written
consent of IMS Learing Resources Pvt.Ltd.and any such violation would entail initiation of suitable legal proceedings.

Copyright

Login ID:1363169/Student Name:SSN/Overall Score:147

1)

2)

3)

4)

5)

iv
at
e

Li
m

ite
d

Explanation:

3)

4)

5)

es

2)

rc

ou

1)

Pr

Question: 98
M % N, N $ P, P ! O
I. M ! O II. O @ N

Le

ar

ni

ng

es

Explanation:

IM

Question: 99
R % S, R $ T, T # U
I. U % S II. R $ U
1)

2)

3)

4)

5)

IMS Learning Resources Pvt.Ltd.,Mumbai.All copyrights to this material vestswith IMS Learning Resources Pvt.Ltd.
No part of this materials either in part oras a whole shall be copied,printed,electronically reproduced,sold or distributed without the written
consent of IMS Learing Resources Pvt.Ltd.and any such violation would entail initiation of suitable legal proceedings.

Copyright

Login ID:1363169/Student Name:SSN/Overall Score:147

ite
d

Explanation:

2)

3)

4)

5)

iv
at
e

1)

Li
m

Question: 100
H ! J, J # I, I $ K
I. J % K II. I @ H

rc

es

Pr

Explanation:

es

ou

Question: 101
Choose the correct alternative.
A retailer sold 6 lemons for Rs. 12 and 2 oranges for Rs. 18. If he suffered 10% loss by selling the oranges but managed to earn 33.33%
profit by selling the lemons, then find his overall percentage profit/loss.
3.45% Profit

2)

3.45% Loss

3)

1.28% Profit

4)

1.28% Loss

5)

No profit no loss

ar

ni

ng

1)

IM

Le

Explanation:

IMS Learning Resources Pvt.Ltd.,Mumbai.All copyrights to this material vestswith IMS Learning Resources Pvt.Ltd.
No part of this materials either in part oras a whole shall be copied,printed,electronically reproduced,sold or distributed without the written
consent of IMS Learing Resources Pvt.Ltd.and any such violation would entail initiation of suitable legal proceedings.

Copyright

iv
at
e

Li
m

ite
d

Login ID:1363169/Student Name:SSN/Overall Score:147

24 cm

3)

16 cm

4)

18 cm

5)

14 cm

rc

2)

ou

12 cm

es

1)

es

Pr

Question: 102

Le

ar

ni

ng

Explanation:

1)

Question: 103
In how many ways can the letters of the word EQUATION be arranged such that no two consonants are together?

2)

10000

3)

14400

4)

720

5)

19600

IM

12400

Explanation:

IMS Learning Resources Pvt.Ltd.,Mumbai.All copyrights to this material vestswith IMS Learning Resources Pvt.Ltd.
No part of this materials either in part oras a whole shall be copied,printed,electronically reproduced,sold or distributed without the written
consent of IMS Learing Resources Pvt.Ltd.and any such violation would entail initiation of suitable legal proceedings.

Copyright

ite
d

Login ID:1363169/Student Name:SSN/Overall Score:147

11 : 7

2)

7 : 11

3)

22 : 49

4)

49 : 22

5)

7:1

iv
at
e

1)

Li
m

Question: 104
If P : Q = 4: 7 and Q : R = 2: 5, what will be the ratio of (P + Q) : (Q + R)?

ou

rc

es

Pr

Explanation:

10

3)

4)

16

5)

12

2)

ng

12

ni

1)

es

Question: 105
If one of the roots of X2 mX + 32 = 0 is 4. Find the value of m.

IM

Le

ar

Explanation:

Question: 106

IMS Learning Resources Pvt.Ltd.,Mumbai.All copyrights to this material vestswith IMS Learning Resources Pvt.Ltd.
No part of this materials either in part oras a whole shall be copied,printed,electronically reproduced,sold or distributed without the written
consent of IMS Learing Resources Pvt.Ltd.and any such violation would entail initiation of suitable legal proceedings.

Copyright

Login ID:1363169/Student Name:SSN/Overall Score:147

Refer to the data below and answer the questions that follow.

20

2)

12

3)

13

4)

18

5)

32

iv
at
e

1)

Li
m

ite
d

All the students of Mahatma Gandhi College are going to participate in one or two or three competitions. The three main competitions are
Singing, Dancing and Elocution.
Total 125 students participated in Singing, 116 in Dancing and 106 in Elocution.
Total 34 students participated in both Dancing and Elocution.
Total 33 students participated in both Singing and Dancing which is also equal to the average of the number of students who participated
in both Singing and Elocution and the number of students who participated in both Dancing and Elocution.
18 students out of the entire college participated in competitions other than the three main ones.
14 students participated in all the three main competitions.
There was no student who participated in a major competition (i.e., Singing, Dancing or Elocution) as well as a competition other than the
three main cities.
How many students participated in both Singing and Elocution but not Dancing?

IM

Hence, [4].

Le

ar

ni

ng

es

ou

rc

es

Pr

Explanation:
Because it is given that the number of students who participated in both Singing and Dancing (33) is equal to the average of the number of
students who participated in both Singing and Elocutionand the number of students who participated in both Dancing and Elocution (34),
therefore the number of students who participated in both Singing and Elocution is 32. Based on the other information given, we can
create the following Venn diagram and then answer all the questions.

Question: 107
What was the total number of students in the college?
1)

260

2)

270

3)

280

IMS Learning Resources Pvt.Ltd.,Mumbai.All copyrights to this material vestswith IMS Learning Resources Pvt.Ltd.
No part of this materials either in part oras a whole shall be copied,printed,electronically reproduced,sold or distributed without the written
consent of IMS Learing Resources Pvt.Ltd.and any such violation would entail initiation of suitable legal proceedings.

Copyright

Login ID:1363169/Student Name:SSN/Overall Score:147

4)

290

5)

300

es

Pr

iv
at
e

Li
m

ite
d

Explanation:
Because it is given that the number of students who participated in both Singing and Dancing (33) is equal to the average of the number of
students who participated in both Singing and Elocution and the number of students who participated in both Dancing and Elocution (34),
therefore the number of students who participated in both Singing and Elocution is 32. Based on the other information given, we can
create the following Venn diagram and then answer all the questions.

rc

Hence, [3].

3)

71

4)

48

5)

58

es

64

2)

ng

57

ni

1)

ou

Question: 108
How many students participated in exactly two competitions from amongst the 3 main competitions (i.e., Singing, Dancing and Elocution)?

IM

Le

ar

Explanation:
Because it is given that the number of students who participated in both Singing and Dancing (33) is equal to the average of the number of
students who participated in both Singing and Elocution and the number of students who participated in both Dancing and Elocution (34),
therefore the number of studentswho participated in both Singing and Elocution is 32. Based on the other information given, we can create
the following Venn diagram and then answer all the questions.

IMS Learning Resources Pvt.Ltd.,Mumbai.All copyrights to this material vestswith IMS Learning Resources Pvt.Ltd.
No part of this materials either in part oras a whole shall be copied,printed,electronically reproduced,sold or distributed without the written
consent of IMS Learing Resources Pvt.Ltd.and any such violation would entail initiation of suitable legal proceedings.

Copyright

Li
m

ite
d

Login ID:1363169/Student Name:SSN/Overall Score:147

iv
at
e

Hence, [1].

2)

58

3)

57

4)

65

5)

54

es

60

rc

1)

Pr

Question: 109
How many students participated in only Elocution?

IM

Le

ar

ni

ng

es

ou

Explanation:
Because it is given that the number of students who participated in both Singing and Dancing (33) is equal to the average of the number of
students who participated in both Singing and Elocution and the number of students who participated in both Dancing and Elocution (34),
therefore the number of students who participated in both Singing and Elocution is 32. Based on the other information given, we can
create the following Venn diagram and then answer all the questions.

Hence, [5].

IMS Learning Resources Pvt.Ltd.,Mumbai.All copyrights to this material vestswith IMS Learning Resources Pvt.Ltd.
No part of this materials either in part oras a whole shall be copied,printed,electronically reproduced,sold or distributed without the written
consent of IMS Learing Resources Pvt.Ltd.and any such violation would entail initiation of suitable legal proceedings.

Copyright

Login ID:1363169/Student Name:SSN/Overall Score:147

Question: 110
How many students participated in exactly one competition from amongst the 3 main competitions (i.e., Singing, Dancing and Elocution)?
1)

188

2)

191

3)

193

4)

190

5)

192

es

ou

rc

es

Pr

iv
at
e

Li
m

ite
d

Explanation:
Because it is given that the number of students who participated in both Singing and Dancing (33) is equal to the average of the number of
students who participated in both Singing and Elocution and the number of students who participated in both Dancing and Elocution (34),
therefore the number of students who participated in both Singing and Elocution is 32. Based on the other information given, we can
create the following Venn diagram and then answer all the questions.

Hence, [2].

Liasion

3)

Liason

4)

Laison

5)

Laision

IM

Explanation:

ni

2)

ar

Liaison

Le

1)

ng

Question: 111
: Directions for questions 111 to 115: Mark the correct spelling.

Liaison means an instance or a means of communication between different groups or units of an organization. Hence, [1].
Question: 112
1)

Perseveerance

2)

Perceiverance

IMS Learning Resources Pvt.Ltd.,Mumbai.All copyrights to this material vestswith IMS Learning Resources Pvt.Ltd.
No part of this materials either in part oras a whole shall be copied,printed,electronically reproduced,sold or distributed without the written
consent of IMS Learing Resources Pvt.Ltd.and any such violation would entail initiation of suitable legal proceedings.

Copyright

Login ID:1363169/Student Name:SSN/Overall Score:147

3)

Perseverance

4)

Percevrance

5)

Percevance

Explanation:

Perseverance means steady persistence in adhering to a course of action, a belief or a purpose. Hence, [3].

2)

Conscientious

3)

Consciencious

4)

Consentious

5)

Conscentious

Li
m

Conscientous

iv
at
e

1)

ite
d

Question: 113

Explanation:

Pr

Conscientious means principled. Hence, [2].

Possesed

3)

Possised

4)

Possessed

5)

Possissed

rc

2)

ou

Posessed

es

1)

es

Question: 114

Explanation:

ng

Possessed means controlled by a strong inner drive. Hence, [4].

2)

Maneuvere

3)

Manoeuvre

4)

Manoeuver

5)

Manaeuver

IM

Explanation:

ar

Manaeuvre

Le

1)

ni

Question: 115

Manoeuvre means a movement or combination of movements involving skill and dexterity. Hence, [3].
Question: 116
Refer to the data below and answer the questions that follow.
There are seven friends K, L, M, N, O, P and Q standing next to each other in a row, all facing north. M is standing exactly in the

IMS Learning Resources Pvt.Ltd.,Mumbai.All copyrights to this material vestswith IMS Learning Resources Pvt.Ltd.
No part of this materials either in part oras a whole shall be copied,printed,electronically reproduced,sold or distributed without the written
consent of IMS Learing Resources Pvt.Ltd.and any such violation would entail initiation of suitable legal proceedings.

Copyright

Login ID:1363169/Student Name:SSN/Overall Score:147

middle of Q and O, and K is standing exactly in the middle of L and N. L is the only person standing to the left of Q.
Who is standing fourth from the extreme left end?
1)

2)

3)

4)

5)

Li
m

Using this, all the questions can be answered.


Hence, [4].

3)

4)

5)

Pr

2)

es

iv
at
e

Question: 117
Who is standing third from the extreme right end?
1)

ite
d

Explanation:
Based on the given conditions, we get only one possibility for the arrangement: L Q K M N O P

ou

rc

Explanation:
Based on the given conditions, we get only one possibility for the arrangement: L Q K M N O P

es

Using this, all the questions can be answered.

ng

Question: 118
What is Ps position from the extreme left end?

Hence, [1].

1st

2)

5th

3)

3rd

4)

7th

5)

Cannot be determined

Le

ar

ni

1)

IM

Explanation:
Based on the given conditions, we get only one possibility for the arrangement: L Q K M N O P
Using this, all the questions can be answered.
Hence, [4].
Question: 119
Who among the following is not standing between L and O?

IMS Learning Resources Pvt.Ltd.,Mumbai.All copyrights to this material vestswith IMS Learning Resources Pvt.Ltd.
No part of this materials either in part oras a whole shall be copied,printed,electronically reproduced,sold or distributed without the written
consent of IMS Learing Resources Pvt.Ltd.and any such violation would entail initiation of suitable legal proceedings.

Copyright

Login ID:1363169/Student Name:SSN/Overall Score:147

1)

2)

3)

4)

5)

ite
d

Explanation:
Based on the given conditions, we get only one possibility for the arrangement: L Q K M N O P
Using this, all the questions can be answered.

Li
m

Hence, [2].

O-P

2)

M-N

3)

Q-M

4)

Q-K

5)

L-Q

Pr

1)

iv
at
e

Question: 120
Which of the following pairs arent immediate neighbours?

rc

es

Explanation:
Based on the given conditions, we get only one possibility for the arrangement: L Q K M N O P

ou

Using this, all the questions can be answered.

es

Hence, [3].

1)

2)

Le

ar

ni

ng

Question: 121
Each question contains a sequence beginning with a diagram labelled [X] and ending with [Y]. Of the five other diagrams labelled [1] to [5],
two adjacent diagrams may have been interchanged. For your answer, identify the two adjacent diagrams that need to be interchanged to
form a logical sequence, and mark the first diagram as your answer. In case no interchange is required, then choose [5].

IM

3)

4)

5)

Explanation:
In alternate steps, rows shift by one position from top to bottom and columns shift by one position from left to right. Therefore, [3] and [4]
need to be interchanged. Hence, [3].

IMS Learning Resources Pvt.Ltd.,Mumbai.All copyrights to this material vestswith IMS Learning Resources Pvt.Ltd.
No part of this materials either in part oras a whole shall be copied,printed,electronically reproduced,sold or distributed without the written
consent of IMS Learing Resources Pvt.Ltd.and any such violation would entail initiation of suitable legal proceedings.

Copyright

Login ID:1363169/Student Name:SSN/Overall Score:147

2)

3)

4)

5)

Li
m

1)

ite
d

Question: 122

iv
at
e

Explanation:

43152

3)

21534

4)

52314

5)

35412

2)

ng

53241

ni

1)

es

ou

rc

es

Question: 123
Choose among the alternatives the combination that will form a logical series.

Pr

In alternate step, the figure rotates 45 clockwise and 90 anticlockwise. In every step, the shading moves one step in the clockwise
direction. Therefore, [4] and [5] need to be interchanged. Hence, [4].

Le

ar

Explanation:
The hexagon moves 90o anticlockwise and the diagonal moves from 1 to 2, 1 to 3, 1 to 4, 1 to 5 and then 1 to 6. So, the correct
arrangement will be 4-3-1-5-2.
Hence, [2].

IM

Question: 124

1)

24351

IMS Learning Resources Pvt.Ltd.,Mumbai.All copyrights to this material vestswith IMS Learning Resources Pvt.Ltd.
No part of this materials either in part oras a whole shall be copied,printed,electronically reproduced,sold or distributed without the written
consent of IMS Learing Resources Pvt.Ltd.and any such violation would entail initiation of suitable legal proceedings.

Copyright

Login ID:1363169/Student Name:SSN/Overall Score:147

2)

41532

3)

14253

4)

53214

5)

25314

ite
d

Explanation:
Number of squares in the top row, number of circles in the middle row and number of triangles in the bottom row reduce by 1 in
successive steps for the arrangement 1-4-2-5-3. Hence, [3].

3)

4)

5)

rc

2)

ou

es

1)

es

Pr

iv
at
e

Li
m

Question: 125
Choose the option that best complete the series.

ni

ng

Explanation:

1)

1017202107

2)

1116192209

3)

Le

ar

Question: 126
Choose the correct alternative.
If PARTY is coded as 1803202227 in a certain code language, then the code for HORSE is

IM

1118212008

4)

1017212007

5)

1017192007

Explanation:

The code for each letter is its position in the alphabet plus 2. P is the 16th letter of the alphabet, so its code is 16 + 2 = 18. Similarly, T is
the 20th letter of the alphabet, so its code is 20 + 2 = 22. H, O, R, S and E are the 8th, 15th, 18th, 19th and 5th letters of the alphabet

IMS Learning Resources Pvt.Ltd.,Mumbai.All copyrights to this material vestswith IMS Learning Resources Pvt.Ltd.
No part of this materials either in part oras a whole shall be copied,printed,electronically reproduced,sold or distributed without the written
consent of IMS Learing Resources Pvt.Ltd.and any such violation would entail initiation of suitable legal proceedings.

Copyright

Login ID:1363169/Student Name:SSN/Overall Score:147

respectively. Therefore, their codes are 10, 17, 20, 21 and 7 respectively. Therefore, the code for HORSE is 1017202107. Hence, [1].

1)

GRVTI

2)

GRITV

3)

GTSVI

4)

GRTVI

5)

GTRVI

ite
d

Question: 127
If CAMEL is coded as XZNVO in a certain code language, then the code for TIGER is

Li
m

Explanation:

iv
at
e

In this case, the alphabetic series is reversed i.e. A is coded as Z, B is coded as Y etc. Therefore the code for TIGER is GRTVI. Hence,
[4].

2)

EJCGO

3)

ANZLK

4)

ECGJO

5)

ENCLO

es

ANYKK

rc

1)

Pr

Question: 128
If HILL is coded as JGNJ and STONE is coded as URQLG in a certain code language, then the code for CLAIM is

ou

Explanation:

es

HILL
+2 -2 +2 -2
JGNJ

Similarly

ni
ar
Le

Therefore
CLAIM
+2 -2 +2 -2 +2
EJCGO
Hence, [2].

ng

STONE
+2 -2 +2 -2 +2
URQLG

IM

Question: 129
If BEAM is coded as DJBZ and DART is coded as HBJN in a certain code language, the code for MOUSE is
1)

MQYWY

2)

ZFRHJ

3)

ZDPJL

4)

MDYLY

5)

ZDPLJ

IMS Learning Resources Pvt.Ltd.,Mumbai.All copyrights to this material vestswith IMS Learning Resources Pvt.Ltd.
No part of this materials either in part oras a whole shall be copied,printed,electronically reproduced,sold or distributed without the written
consent of IMS Learing Resources Pvt.Ltd.and any such violation would entail initiation of suitable legal proceedings.

Copyright

Login ID:1363169/Student Name:SSN/Overall Score:147

Explanation:
The code for each letter is the letter of the alphabet at the position equal to twice the position of the given letter. For example, B is the 2nd
letter of the alphabet. Twice of 2 is 4. D is the 4th letter of the alphabet. Therefore the code of B is D. Similarly other codes are arrived at. If
the product exceeds 26, position of the code is equal to the product minus 26. For example, R is 18th letter. Twice of R is 36. Therefore
the position of the code is 10th in the alphabet (J).
MOUSE are 13th, 15th, 21st, 19th and 5th letters of the alphabet. Therefore the positions of their codes are 26th, 4th, 16th, 12th and 10th or
the required code is ZDPLJ. Hence, [5].

2)

AGFIMNY

3)

AFGIMNY

4)

AFIGNMY

5)

AFGMINY

Li
m

AFGINMY

iv
at
e

1)

ite
d

Question: 130
If GREAT is coded as AEGRT is a certain code language, the code for MAGNIFY is

Explanation:

Pr

The letters are arranged in alphabetical order. Therefore the code for MAGNIFY is AFGIMNY. Hence, [3].

Heifer

2)

Boar

3)

Ewe

4)

Filly

5)

Mare

es

ou

rc

1)

es

Question: 131
Choose the odd one out.

ng

Explanation:

All except boar are names for the female gender of different animals. A boar is a male pig. Hence, [2].

Iris

2)

Pupil

3)

Cornea

4)

Retina

5)

Le

ar

1)

ni

Question: 132

IM

Cochlea

Explanation:

All except [5] are parts of the eye. Cochlea is a part of the ear. Hence, [5].
Question: 133
1)

Boa

IMS Learning Resources Pvt.Ltd.,Mumbai.All copyrights to this material vestswith IMS Learning Resources Pvt.Ltd.
No part of this materials either in part oras a whole shall be copied,printed,electronically reproduced,sold or distributed without the written
consent of IMS Learing Resources Pvt.Ltd.and any such violation would entail initiation of suitable legal proceedings.

Copyright

Login ID:1363169/Student Name:SSN/Overall Score:147

2)

Basa

3)

Cobra

4)

Python

5)

Viper

Explanation:

ite
d

Basa is a fish. All others are types of snakes. Hence, [2].

Bow

2)

Hull

3)

Mast

4)

Stern

5)

Troll

iv
at
e

1)

Li
m

Question: 134

Pr

Explanation:

All except troll are parts of a boat. Troll is a fishing technique, which means to tow a lure or bait behind a moving boat. Hence, [5].

Quilting

3)

Papier mch

4)

Origami

5)

Quilling

rc

2)

ou

Paper filigree

es

1)

es

Question: 135

ng

Explanation:

ni

All except [2] are decorative arts using paper. Quilting is a technique of sewing. Hence, [2].

2)

Pucker

3)

Furrow

4)

Crinkle

5)

ar

Wrinkle

Le

1)

Question: 136

IM

Mangle

Explanation:

Wrinkle, pucker, furrow and crinkle are facial contortions, while mangle refers to total mutilation by cutting or battering. Hence, [5].
Question: 137
1)

Dentist

IMS Learning Resources Pvt.Ltd.,Mumbai.All copyrights to this material vestswith IMS Learning Resources Pvt.Ltd.
No part of this materials either in part oras a whole shall be copied,printed,electronically reproduced,sold or distributed without the written
consent of IMS Learing Resources Pvt.Ltd.and any such violation would entail initiation of suitable legal proceedings.

Copyright

Login ID:1363169/Student Name:SSN/Overall Score:147

2)

Ophthalmologist

3)

Podiatrist

4)

Philatelist

5)

Chiropodist

Explanation:
All except philatelist are medical specialists treating particular body parts. A philatelist collects or studies stamps. Hence, [4].

2)

Cherry

3)

Pear

4)

Carrot

5)

Watermelon

Li
m

Apple

iv
at
e

1)

ite
d

Question: 138

Explanation:

Pr

A carrot is a vegetable. All others are fruits. Hence, [4].

2)

Cufflink

3)

Socks

4)

Shirt

5)

Belt

rc

Tie

es

ou

1)

es

Question: 139

Explanation:

ng

All except shirt are accessories for mens clothing. Hence, [4].

Hue

2)

Pigment

3)

Motif

4)

Shade

5)

Tint

Le

ar

1)

ni

Question: 140

IM

Explanation:
Hue, shade, tint and pigment are words associated with colours. However, motif, which means primary theme, is associated with
music or literature. Hence, [3].
Question: 141
Choose the correct alternative.
Five people P, Q, R, S and T are standing according to their height. The number of people shorter than Q is equal to the number of people
taller than S. P is not the shortest and R is not the tallest. If T is the tallest, then who is the 3rd tallest person?
1)

IMS Learning Resources Pvt.Ltd.,Mumbai.All copyrights to this material vestswith IMS Learning Resources Pvt.Ltd.
No part of this materials either in part oras a whole shall be copied,printed,electronically reproduced,sold or distributed without the written
consent of IMS Learing Resources Pvt.Ltd.and any such violation would entail initiation of suitable legal proceedings.

Copyright

Login ID:1363169/Student Name:SSN/Overall Score:147

2)

3)

4)

5)

Cannot be determined

Explanation:

Li
m

Question: 142
Refer to the data below and answer the questions that follow.

ite
d

Consider tallest person is standing at the leftmost position i.e., following combinations are possible.
T Q/S P S/Q R
Hence, P is the third tallest person. Hence, [4].

3)

V and S

4)

R and S

5)

P and S

es

W and U

rc

2)

ou

W and T

es

1)

Pr

iv
at
e

Around a square-shaped table, 8 people P, Q, R, S, T, U, V and W are sitting facing the centre such that the distance between any 2
adjacent people along the edge of the table is the same. The following information is also known.
1. 4 people sit at corner positions and 4 people at the centre of the sides of the table.
2. Along the edge of the table, V is 2 places to the right of S, who is not sitting at a corner position.
3. U is sitting directly opposite T.
4. W is the only person between P and R along the edge of the table.
Q is the only person who sits between:

IM

Le

ar

ni

ng

Explanation:

IMS Learning Resources Pvt.Ltd.,Mumbai.All copyrights to this material vestswith IMS Learning Resources Pvt.Ltd.
No part of this materials either in part oras a whole shall be copied,printed,electronically reproduced,sold or distributed without the written
consent of IMS Learing Resources Pvt.Ltd.and any such violation would entail initiation of suitable legal proceedings.

Copyright

es

ou

rc

es

Pr

iv
at
e

Li
m

ite
d

Login ID:1363169/Student Name:SSN/Overall Score:147

Q sits between V and S. Hence, [3].

ng

Question: 143
If P is the only person who sits between W and T, then who sits immediate to the right of V?
T

2)

3)

4)

5)

Cannot be determined

Le

ar

ni

1)

IM

Explanation:

IMS Learning Resources Pvt.Ltd.,Mumbai.All copyrights to this material vestswith IMS Learning Resources Pvt.Ltd.
No part of this materials either in part oras a whole shall be copied,printed,electronically reproduced,sold or distributed without the written
consent of IMS Learing Resources Pvt.Ltd.and any such violation would entail initiation of suitable legal proceedings.

Copyright

es

ou

rc

es

Pr

iv
at
e

Li
m

ite
d

Login ID:1363169/Student Name:SSN/Overall Score:147

In this case, P can sit either to the immediate left or the immediate right of W. So, either T or U can sit immediate to the right of V. Hence,
[5].

ng

Question: 144
If T sits adjacent to V, then U will sit immediate to the left of ____.
R

2)

3)

4)

5)

None of these

ar
Le

IM

Explanation:

ni

1)

IMS Learning Resources Pvt.Ltd.,Mumbai.All copyrights to this material vestswith IMS Learning Resources Pvt.Ltd.
No part of this materials either in part oras a whole shall be copied,printed,electronically reproduced,sold or distributed without the written
consent of IMS Learing Resources Pvt.Ltd.and any such violation would entail initiation of suitable legal proceedings.

Copyright

es

ou

rc

es

Pr

iv
at
e

Li
m

ite
d

Login ID:1363169/Student Name:SSN/Overall Score:147

If T sits adjacent to V, then S will sit immediate to the left of U. Hence, [3].

ng

Question: 145
If R is to the immediate left of T, then who will be sitting directly opposite V?
P

2)

3)

4)

5)

Cannot be determined

Le

ar

ni

1)

IM

Explanation:

IMS Learning Resources Pvt.Ltd.,Mumbai.All copyrights to this material vestswith IMS Learning Resources Pvt.Ltd.
No part of this materials either in part oras a whole shall be copied,printed,electronically reproduced,sold or distributed without the written
consent of IMS Learing Resources Pvt.Ltd.and any such violation would entail initiation of suitable legal proceedings.

Copyright

es

ou

rc

es

Pr

iv
at
e

Li
m

ite
d

Login ID:1363169/Student Name:SSN/Overall Score:147

If R is to the immediate left of U, then R will be sitting opposite V. Hence, [4].

ng

Question: 146
If V is facing North, then who is facing South-West?
T

2)

3)

4)

5)

Cannot be determined

ar
Le

IM

Explanation:

ni

1)

IMS Learning Resources Pvt.Ltd.,Mumbai.All copyrights to this material vestswith IMS Learning Resources Pvt.Ltd.
No part of this materials either in part oras a whole shall be copied,printed,electronically reproduced,sold or distributed without the written
consent of IMS Learing Resources Pvt.Ltd.and any such violation would entail initiation of suitable legal proceedings.

Copyright

es

If V is facing North, W is facing South-West. Hence, [4].

ou

rc

es

Pr

iv
at
e

Li
m

ite
d

Login ID:1363169/Student Name:SSN/Overall Score:147

Question: 147
For the previous question, R and P are facing _______.
South and West

2)

South and East

3)

North East and South East

4)

North West and South

5)

North East and South

Le

ar

ni

ng

1)

IM

Explanation:

IMS Learning Resources Pvt.Ltd.,Mumbai.All copyrights to this material vestswith IMS Learning Resources Pvt.Ltd.
No part of this materials either in part oras a whole shall be copied,printed,electronically reproduced,sold or distributed without the written
consent of IMS Learing Resources Pvt.Ltd.and any such violation would entail initiation of suitable legal proceedings.

Copyright

ou

rc

es

Pr

iv
at
e

Li
m

ite
d

Login ID:1363169/Student Name:SSN/Overall Score:147

es

R and P will be facing South and West (in no specific order). Hence, [1].

3)

4)

49

5)

121

ng

169

ni

2)

ar

Le

1)

Question: 148
Find the odd term in the series.
1, 9, 25, 49, 121, 169

Explanation:

IM

The series is a series of squares of prime numbers.


22, 32, 52, 72, 112, 132.Hence, [1].
Question: 149
Choose the correct alternative.
Four of the five numbers are alike in a certain way and so form a group. Which is the one that does not belong to the group?
1)

1728

2)

2197

IMS Learning Resources Pvt.Ltd.,Mumbai.All copyrights to this material vestswith IMS Learning Resources Pvt.Ltd.
No part of this materials either in part oras a whole shall be copied,printed,electronically reproduced,sold or distributed without the written
consent of IMS Learing Resources Pvt.Ltd.and any such violation would entail initiation of suitable legal proceedings.

Copyright

Login ID:1363169/Student Name:SSN/Overall Score:147

3)

2744

4)

1000

5)

4056

Explanation:

All of these numbers are perfect cubes except 4056. Hence, [5].

2)

48

3)

122

4)

168

5)

195

Li
m

24

iv
at
e

1)

ite
d

Question: 150
Find the odd term in the series: 3, 8, 24, 48, 122, 168, 288

Explanation:

es

Pr

The series is of the formPrime Number2 1. Therefore, the series is 22 1, 32 1, 52 1, 72 1, 112 1, 132 1 and 172 1.
Therefore, the series is 3, 8, 24, 48, 120, 168, 288. Therefore, the odd term is 122. Hence, [3].

rc

Question: 151
Choose the option which correctly converts the given sentence into passive voice.
The rain is washing the soil away faster than expected.
Soil is washed away by the rain faster than can be expected.

2)

Soil has been washed away by the rain faster than expected.

3)

Soil began to be washed away by the rain faster than expected.

4)

Soil is washed away faster than is expected by the rain.

5)

Soil is being washed away by the rain faster than expected.

ng

es

ou

1)

ni

Explanation:

Le

ar

Since the sentence is in the present continuous tense, the tense in the passive voice will be is being washed. [5] follows this rule
correctly. Hence, [5].
Question: 152
In Nepal, monocropping destroyed most of the original forests which supported the human population.
The human population in Nepal supporting the original forests were destroyed by monocropping.

2)

1)

3)

In Nepal, most of the original forests which supported the human population was destroyed by monocropping.

4)

Most of the original forests which supported monocropping in Nepal was destroyed by human population.

5)

Most of the original forests which Nepal supported were destroyed by monocropping.

IM

In Nepal, most of the original forests which supported the human population were destroyed by monocropping.

Explanation:

IMS Learning Resources Pvt.Ltd.,Mumbai.All copyrights to this material vestswith IMS Learning Resources Pvt.Ltd.
No part of this materials either in part oras a whole shall be copied,printed,electronically reproduced,sold or distributed without the written
consent of IMS Learing Resources Pvt.Ltd.and any such violation would entail initiation of suitable legal proceedings.

Copyright

Login ID:1363169/Student Name:SSN/Overall Score:147

[1], [4] and [5] completely change the meaning of the sentence. Thus they can be negated. [3] wrongly uses the singular verb was
instead of were. Only [2] converts the sentence in the passive voice correctly. Hence, [2].

1)

A research vessel bound for a part of the Indian Ocean was boarded by scientists.

2)

A research vessel Indian-Ocean bound was boarded by scientists.

3)

A research vessel bound for a part of the Indian Ocean were boarded by scientists.

4)

A research vessel bound for a part of the Indian Ocean was boarded by scientist.

5)

A research vessel was boarded by scientists bound for a part of the Indian Ocean.

ite
d

Question: 153
Scientists boarded a research vessel bound for a part of the Indian Ocean.

Li
m

Explanation:

Question: 154
Humans cultivated peaches much before other fruits.
Peaches were cultivated by humans much before cultivating other fruits.

2)

Peaches were much cultivated by humans before other fruits.

3)

Peaches got cultivated by humans much before they did other fruits.

4)

Peaches were cultivated by humans much before other fruits.

5)

Peaches were being cultivated by humans before other fruits were.

rc

es

Pr

1)

iv
at
e

To convert a sentence in active voice into passive voice, the object in the active voice is moved to the subject position. While [1], [3] and
[4] follow this rule, [3] uses the wrong form of the verb (were) and [4] uses the singular form scientist. [2] and [5] can be negated as they
distort the sentence structure. [1] presents the sentence in passive form correctly. Hence, [1].

ou

Explanation:

es

[1] and [3] unnecessarily reframe the sentence. [2] wrongly places much, thus changing the meaning of the sentence. [5] is incorrect as it
does not maintain tense consistency; it uses the past continuous tense instead of the past tense. Only [4] converts the sentence into its
passive form correctly. Hence, [4].

ng

Question: 155
A disease-resistant variety known as the Cavendish saved the banana industry.
The banana was saved by a disease-resistant variety known as the Cavendish.

2)

The Cavendish banana was a disease-resistant variety saved by the industry.

3)

The banana industry was saved by a disease-resistant variety known as the Cavendish.

4)

The industry was saved by a disease-resistant variety known as the Cavendish.

5)

A disease-resistant variety of Cavendish was saved by the banana industry.

ar

Le

IM

Explanation:

ni

1)

Only [3] converts the sentence into passive voice correctly. All others distort the meaning of the sentence in the passive form. Hence, [3].
Question: 156
Following table shows the total power generated (in Lakh units) using wind energy and solar energy in 2014 and 2015 in
different states of India. Based on the table, answer the questions that follow.

IMS Learning Resources Pvt.Ltd.,Mumbai.All copyrights to this material vestswith IMS Learning Resources Pvt.Ltd.
No part of this materials either in part oras a whole shall be copied,printed,electronically reproduced,sold or distributed without the written
consent of IMS Learing Resources Pvt.Ltd.and any such violation would entail initiation of suitable legal proceedings.

Copyright

Madhya Pradesh

2)

Rajasthan

3)

Karnataka

4)

Uttar Pradesh

5)

Gujarat

Pr

1)

iv
at
e

Which state has shown maximum percent increase in wind power generated between 2014 and 2015?

Li
m

ite
d

Login ID:1363169/Student Name:SSN/Overall Score:147

es

Explanation:

ou

rc

States of Karnataka, Rajasthan, Uttar Pradesh, Gujarat and Madhya Pradesh registered an increase in wind power generated in 2015
over 2014. Out of these states, only Rajasthan generated more than double wind power in 2015 than in 2014. Hence, [2].

es

Question: 157
If the states in which solar power generated decreased between 2014 and 2015 were to be ordered in the descending order of the percent
decrease in solar power generated in 2015 over in 2014, which state would be ranked second?
Tamil Nadu

2)

Punjab

3)

Uttar Pradesh

4)

Bihar

5)

None of the above

ar

ni

ng

1)

IM

Le

Explanation:

IMS Learning Resources Pvt.Ltd.,Mumbai.All copyrights to this material vestswith IMS Learning Resources Pvt.Ltd.
No part of this materials either in part oras a whole shall be copied,printed,electronically reproduced,sold or distributed without the written
consent of IMS Learing Resources Pvt.Ltd.and any such violation would entail initiation of suitable legal proceedings.

Copyright

iv
at
e

Li
m

ite
d

Login ID:1363169/Student Name:SSN/Overall Score:147

2)

Gujarat

3)

Madhya Pradesh

4)

Uttar Pradesh

5)

Jharkhand

es

Rajasthan

rc

1)

Pr

Question: 158
Which of the following states has exhibited maximum increase in total energy generated (wind and solar) between 2014 and 2015?

Le

ar

ni

ng

es

ou

Explanation:

IM

Question: 159
What is the percent increase in the total solar power generated in 2015 over the total solar power generated in 2014 for all of the given
states?
1)

4%

2)

3%

3)

6%

4)

15%

IMS Learning Resources Pvt.Ltd.,Mumbai.All copyrights to this material vestswith IMS Learning Resources Pvt.Ltd.
No part of this materials either in part oras a whole shall be copied,printed,electronically reproduced,sold or distributed without the written
consent of IMS Learing Resources Pvt.Ltd.and any such violation would entail initiation of suitable legal proceedings.

Copyright

Login ID:1363169/Student Name:SSN/Overall Score:147

5)

9%

rc

es

Pr

iv
at
e

Li
m

ite
d

Explanation:

5.8 Lakh units

3)

6.2 Lakh units

4)

5.5 Lakh units

5)

6.5 Lakh units

IM

Le

ar

ni

Explanation:

es

2)

6 Lakh units

ng

1)

ou

Question: 160
What is the average wind power generated in 2014 in all the given states?

IMS Learning Resources Pvt.Ltd.,Mumbai.All copyrights to this material vestswith IMS Learning Resources Pvt.Ltd.
No part of this materials either in part oras a whole shall be copied,printed,electronically reproduced,sold or distributed without the written
consent of IMS Learing Resources Pvt.Ltd.and any such violation would entail initiation of suitable legal proceedings.

Copyright

iv
at
e

Li
m

ite
d

Login ID:1363169/Student Name:SSN/Overall Score:147

es

ou

rc

es

Pr

Question: 161
Each question given below consists of a statement, followed by two arguments numbered I and II. Based on the given
statement, you have to decide if the given arguments are strong or weak.
Mark [a] if only I is strong,
Mark [b] if only II is strong,
Mark [c] if both I and II are strong,
Mark [d] if neither I nor II is strong, and
Mark [e] if either I or II is strong
Statement: China should end its decades-long one-child policy, and allow all couples to have two children.
Arguments:
I. Yes: Chinas population is ageing rapidly, gender imbalances are severe and its workforce is shrinking.
II. No: For years, the one-child policy has contributed to Chinas economic boom by keeping population growth in check.
if only I is strong,

2)

if only II is strong,

3)

if both I and II are strong,

4)

if neither I nor II is strong, and

5)

if either I or II is strong

ar

ni

ng

1)

Le

Explanation:

IM

Both I and II are strong arguments as they give valid reasons for and against rescinding Chinas one child policy respectively. Hence, [3].
Question: 162
Statement: The telecom regulators order that carriers must compensate users for dropped calls is unjust and harsh.
Arguments:
I.Yes: It could incentivize subscribers to drop calls in order to receive compensation.
II.No: The telecom industry has been aiding delivery of voice and data services at high speeds.
1)

if only I is strong,

2)

if only II is strong,

IMS Learning Resources Pvt.Ltd.,Mumbai.All copyrights to this material vestswith IMS Learning Resources Pvt.Ltd.
No part of this materials either in part oras a whole shall be copied,printed,electronically reproduced,sold or distributed without the written
consent of IMS Learing Resources Pvt.Ltd.and any such violation would entail initiation of suitable legal proceedings.

Copyright

Login ID:1363169/Student Name:SSN/Overall Score:147

3)

if both I and II are strong,

4)

if neither I nor II is strong, and

5)

if either I or II is strong

Explanation:

ite
d

I is a strong argument, as it states that subscribers may deliberately drop calls, which will impact the carriers negatively. II is a general
statement that does not affect the argument in any way. Hence, [1].

if only I is strong,

2)

if only II is strong,

3)

if both I and II are strong,

4)

if neither I nor II is strong, and

5)

if either I or II is strong

Pr

1)

iv
at
e

Li
m

Question: 163
Statement: The government should enter into an agreement with the Ministry of Defence that will allow land under its control to be used
for mass housing.
Arguments:
I.Yes: The government should be working on more plans for mass housing.
II.No: This will concretize the city, and deprive people of the well preserved defence lands which are the only lung spaces left in the city.

rc

es

Explanation:

ou

I just a vague and general assertion, not an argument at all, so it is weak. II is a strong argument, as it is a cogent argument against the
use of defence land for mass housing projects. Hence, [2].

ng

es

Question: 164
Statement: Nuclear energy is a clean energy option, and therefore a solution to climate change.
Arguments:
I.Yes: Indias civilian nuclear programme has been indigenous for many years, but the government is now considering foreign investment.
II.No: The nuclear fuel cycle produces radioactive waste that causes accidents and pollutes the environment for generations.
if only I is strong,

2)

if only II is strong,

3)

if both I and II are strong,

4)

if neither I nor II is strong, and

5)

if either I or II is strong

Le

ar

ni

1)

IM

Explanation:

I is a weak argument as it does not address the issue of nuclear energy as a solution to climate change. II is a strong argument as it
contradicts the idea that nuclear energy is clean energy with a valid reason. Hence, [2].
Question: 165
Statement: The Environment Ministry should mandate all textile firms in India to reduce their effluent discharge to zero.
Arguments:
I.Yes: One must ensure best practices in all important sectors of the Indian economy.
II.No: It would make Indian firms even more uncompetitive at a time when export orders are shrinking.

IMS Learning Resources Pvt.Ltd.,Mumbai.All copyrights to this material vestswith IMS Learning Resources Pvt.Ltd.
No part of this materials either in part oras a whole shall be copied,printed,electronically reproduced,sold or distributed without the written
consent of IMS Learing Resources Pvt.Ltd.and any such violation would entail initiation of suitable legal proceedings.

Copyright

Login ID:1363169/Student Name:SSN/Overall Score:147

1)

if only I is strong,

2)

if only II is strong,

3)

if both I and II are strong,

4)

if neither I nor II is strong, and

5)

if either I or II is strong

ite
d

Explanation:

Li
m

I is a weak argument, as it is a generalized statement about best practices. II is a strong argument as it presents a possible consequence
of the mandate.
Hence, [2].

3)

4)

5)

ou

2)

es

1)

rc

es

Pr

iv
at
e

Question: 166
Each question is followed by two statements I and II.
Mark [a], if the question can be answered by using statement I alone.
Mark [b], if the question can be answered by using statement II alone.
Mark [c], if the question can be answered by using either of the statements alone.
Mark [d], if the question can be answered by using both the statements together, but cannot be answered by using either of the
statements alone.
Mark [e], if more data is required to answer the question.
There are seven benches in a garden numbered 1 to 7 from left to right in that order. Each bench is occupied by one out of A, B, C, D and
E. Benches 2 and 5 are lying vacant. On which bench does D sit?
I. C sits on an even-numbered bench. D sits on a bench to the immediate left of C.
II. D sits 2 benches to the right of A, who sits on an odd-numbered bench.

ng

Explanation:

Le

ar

ni

From statement I, C sits on an even-numbered bench, so he can sit on the 4th or 6th bench (as bench 2 is vacant). Also, since D sits on a
bench to the immediate left of C, C can sit only on the 4th bench as the 5th bench is vacant. So, D sits on the 3rd bench. Statement I
alone is sufficient to answer the question.
From statement II, as the 5th bench is unoccupied, neither A nor D can sit on the 5th bench. So, the only possible combination for A and D
is to sit on the 1st and 3rd benches respectively. Statement II alone can answer the question.
Therefore, either statement alone can answer the question independently.
Hence, [3].

IM

Question: 167
In a family of 6 people (P, Q, R, S, T and U) across 2 generations, all are married. R and S are sisters and one of them is married to U.
How is T related to P?
I. One of the parents of R is P.
II. T is brother-in-law of S.
1)

2)

3)

IMS Learning Resources Pvt.Ltd.,Mumbai.All copyrights to this material vestswith IMS Learning Resources Pvt.Ltd.
No part of this materials either in part oras a whole shall be copied,printed,electronically reproduced,sold or distributed without the written
consent of IMS Learing Resources Pvt.Ltd.and any such violation would entail initiation of suitable legal proceedings.

Copyright

Login ID:1363169/Student Name:SSN/Overall Score:147

4)

5)

Explanation:

Li
m

ite
d

From statement I, family tree is as below:

iv
at
e

Now in the first generation, we know one of the people is P, but we do not know the gender of P. Also we do not know who amongst T and
Q is married to P. So this statement alone is not enough.
From statement II, we know T is married to R and U to S. We do not know who amongst P and Q is the father and mother. But T will be
son-in-law of P as well as Q. (as both are parents of R and S). So statement II alone is sufficient to answer the question. Hence, [2].

3)

4)

5)

rc

2)

ou

es

1)

es

Pr

Question: 168
On a circular table, J, K, L, M, N and O sit facing the centre of the table in such a way that each person is sitting equidistant from the 2
people sitting adjacent to him/her. N is sitting to the immediate left of J and K is sitting to the immediate right of M. Who is sitting directly
opposite O?
I. K is sitting equidistant from J and L.
II. K is sitting adjacent to N.

IM

Le

ar

ni

ng

Explanation:

IMS Learning Resources Pvt.Ltd.,Mumbai.All copyrights to this material vestswith IMS Learning Resources Pvt.Ltd.
No part of this materials either in part oras a whole shall be copied,printed,electronically reproduced,sold or distributed without the written
consent of IMS Learing Resources Pvt.Ltd.and any such violation would entail initiation of suitable legal proceedings.

Copyright

IM

Le

ar

ni

ng

es

ou

rc

es

Pr

iv
at
e

Li
m

ite
d

Login ID:1363169/Student Name:SSN/Overall Score:147

Question: 169
In a particular coding system, My name is Lallu is written as ja ka la pa and Lallu is from Bihar is written as ha ka ma pa what is the
code for is?
I. Bihar is famous is written as ka ma za.
II. Lallu is famous is written as ka pa za.
1)

IMS Learning Resources Pvt.Ltd.,Mumbai.All copyrights to this material vestswith IMS Learning Resources Pvt.Ltd.
No part of this materials either in part oras a whole shall be copied,printed,electronically reproduced,sold or distributed without the written
consent of IMS Learing Resources Pvt.Ltd.and any such violation would entail initiation of suitable legal proceedings.

Copyright

Login ID:1363169/Student Name:SSN/Overall Score:147

2)

3)

4)

5)

Explanation:

Li
m

ite
d

From given data we know that code for Lallu and is pa and ka (in no specific order).
From statement I and the given data, only is is common, so we can determine that the code of is is ka. Statement I alone is sufficient
to answer the question.
From given data and statement we do not get additional information about codes for Lallu and is. So statement II alone is not sufficient.
Hence, [1].

2)

3)

4)

5)

Pr

es

1)

iv
at
e

Question: 170
The shortest possible distance between P and Q is 25 km. What is the direction of point P with respect to point Q?
I. The shortest possible distance between P and a point R is 7 km.
II. To travel from a point R to point Q, the shortest possible distance one has to travel is 24 km in the South direction.

IM

Le

ar

ni

ng

es

ou

rc

Explanation:

Question: 171
Each question contains a series beginning with the diagram labelled [X] and ending with [Y]. Out of the five figures marked 1 to 5, find the
one which does not fit in the series.

IMS Learning Resources Pvt.Ltd.,Mumbai.All copyrights to this material vestswith IMS Learning Resources Pvt.Ltd.
No part of this materials either in part oras a whole shall be copied,printed,electronically reproduced,sold or distributed without the written
consent of IMS Learing Resources Pvt.Ltd.and any such violation would entail initiation of suitable legal proceedings.

Copyright

2)

3)

4)

5)

Li
m

1)

ite
d

Login ID:1363169/Student Name:SSN/Overall Score:147

Explanation:

iv
at
e

The circle moves one step clockwise in every step. The star moves diagonally in each step. In box [2], instead of moving clock-wise, circle
moves in the anti-clockwise direction. Hence, [2].

3)

4)

5)

ou

2)

es

1)

rc

es

Pr

Question: 172

ni

ng

Explanation:

ar

By observing box [Y] we get that, the top line of the square vanishes after the bottom line vanishes. But this is not followed in box [2].
Hence, [2].

IM

Le

Question: 173

1)

2)

3)

4)

IMS Learning Resources Pvt.Ltd.,Mumbai.All copyrights to this material vestswith IMS Learning Resources Pvt.Ltd.
No part of this materials either in part oras a whole shall be copied,printed,electronically reproduced,sold or distributed without the written
consent of IMS Learing Resources Pvt.Ltd.and any such violation would entail initiation of suitable legal proceedings.

Copyright

Login ID:1363169/Student Name:SSN/Overall Score:147

5)

ite
d

Explanation:

2)

3)

4)

5)

Pr

es

1)

iv
at
e

Li
m

Question: 174

rc

Explanation:

es

ou

For the squares at upper left and the lower right corners, the shaded part moves diagonally for every step. For the squares at lower left
and the upper right the shaded part moves one step clockwise in every step. But in box [3], the shaded part of square in lower left moves
in anti-clockwise direction. Hence, [3].

Le

ar

ni

ng

Question: 175
Each question below contains a master figure divided into four sections, one of which is missing. Choose from options [a) to [e)
to best complete the master figure.

2)

1)

3)

4)

5)

IM

Explanation:
Hence, [4].

IMS Learning Resources Pvt.Ltd.,Mumbai.All copyrights to this material vestswith IMS Learning Resources Pvt.Ltd.
No part of this materials either in part oras a whole shall be copied,printed,electronically reproduced,sold or distributed without the written
consent of IMS Learing Resources Pvt.Ltd.and any such violation would entail initiation of suitable legal proceedings.

Copyright

Login ID:1363169/Student Name:SSN/Overall Score:147

1)

Banu Tanu Panu

2)

Banu Danu Panu

3)

Canu Danu Hanu

4)

Canu Hanu Tanu

5)

Banu Hanu Panu

ite
d

Question: 176
Choose the correct alternative.
In a code language, Banu Tanu Hanu means I am tall, Canu Danu Hanu means He is tall, and Canu Danu Panu means He is
thin. Find the code for I am thin

Li
m

Explanation:

Banu Tanu Hanu means I am tall. Canu Danu Hanu means He is tall. Therefore Hanu means tall and Banu Tanu means I am.

iv
at
e

Canu Danu Hanu means He is tall, out of which Hanu means tall. Therefore Canu Danu means He is. Further, Canu Danu Panu
means He is thin. Therefore the code for thin is Panu.
Therefore I am thin is coded as Banu Tanu Panu.
Hence, [1].

639

3)

549

4)

459

5)

369

rc

2)

ou

279

es

1)

es

Pr

Question: 177
In a code language, SINGULAR is coded as 538, PLURAL is coded as 426, PRIMARY is coded as 527 and SKY is coded as 303.
Find the code for IMPORTANT

Explanation:

ni

ng

The first digit of the code is the number of consonants, the second digit of the code is the number of vowels while the third digit of the code
is total number of letters in the word. Therefore the code for IMPORTANT is 639. Hence, [2].

1)

Ja Nu wa te ft Pe Mr of Ia

2)

Jl Nu wa te ft Pe Mr of Ia

3)

Le

ar

Question: 178
In a code language, the code for Barrack Obama is the President of America is Bk Oa is te Pt of Aa and the code for David Cameron
is the Prime Minister of England is Dd Cn is te Pe Mr of Ed. Find the code for Jawaharlal Nehru was the first Prime Minister of India

IM

Jl Nr ws te ft Pe Mr of Ia

4)

Jl Nu ws te ft Pr Mr of Ia

5)

Jl Nu ws te ft Pe Mr of Ia

Explanation:

The code for each word is the first and the last letter of the word. Therefore required code for Jawaharlal Nehru was the first Prime
Minister of India is Jl Nu ws te ft Pe Mr of Ia. Hence, [5].

IMS Learning Resources Pvt.Ltd.,Mumbai.All copyrights to this material vestswith IMS Learning Resources Pvt.Ltd.
No part of this materials either in part oras a whole shall be copied,printed,electronically reproduced,sold or distributed without the written
consent of IMS Learing Resources Pvt.Ltd.and any such violation would entail initiation of suitable legal proceedings.

Copyright

Login ID:1363169/Student Name:SSN/Overall Score:147

1)

FOUSPQZ

2)

FMUQPOZ

3)

DMSQNOX

4)

DOSSNQX

5)

DMUQPOX

ite
d

Question: 179
In a code language, the code for ROBOTICS is SNCNUHDR and the code for ENGINE is FMHHOD. Find the code for ENTROPY

Li
m

Explanation:

The code contains the next letter for all letters in odd positions and the previous letter for all letters in even positions. Therefore the code
for ENTROPY is FMUQPOZ.Hence, [2].

2)

WDIHDKF

3)

UDIJBMF

4)

WFGJDMF

5)

UFGJBKD

Pr

UFGJBMD

es

1)

iv
at
e

Question: 180
In a code language, the code for PLANE is OMZOD and the code for TRAIN is SSZJM. Find the code for VEHICLE

ou

rc

Explanation:

es

The code contains the previous letter for all letters in odd positions and the next letter for all letters in even positions. Therefore the code
for VEHICLE is UFGJBMD.Hence, [1].

Invalidate

3)

Institute

4)

Revoke

5)

Challenge

ni

2)

ar

Annihilate

Le

1)

ng

Question: 181
Choose the correct antonyms from the given options.
Vitiate

Explanation:

IM

Vitiate means invalidate. Thus, institute is the antonym. All the other words are synonyms or near-synonyms of the word. Hence, [3].
Question: 182
Decry
1)

Discredit

2)

Digress

3)

Exalt

IMS Learning Resources Pvt.Ltd.,Mumbai.All copyrights to this material vestswith IMS Learning Resources Pvt.Ltd.
No part of this materials either in part oras a whole shall be copied,printed,electronically reproduced,sold or distributed without the written
consent of IMS Learing Resources Pvt.Ltd.and any such violation would entail initiation of suitable legal proceedings.

Copyright

Login ID:1363169/Student Name:SSN/Overall Score:147

4)

Exaggerate

5)

Undervalue

Explanation:
Decry means to express disapproval of (a person). Exalt is the antonym. Discredit and undervalue are synonyms. Exaggerate and
digress are not related to decry. Hence, [3].

Agile

2)

Lethargic

3)

Pensive

4)

Evasive

5)

Methodical

Li
m

1)

ite
d

Question: 183
Sprightly

iv
at
e

Explanation:

The antonym of sprightly (which means lively) is lethargic (which means lazy or sluggish). Hence, [2].

Detached

3)

Prejudiced

4)

Unmediated

5)

Wayward

rc

2)

ou

Detailed

es

1)

es

Pr

Question: 184
Perfunctory

Explanation:

Energetic

2)

Malevolent

3)

Favourable

4)

Le

1)

Question: 185
Baleful

ar

ni

ng

Perfunctory means done superficially, only as a matter of routine. [2] and [4] are synonyms. [3] and [5] are not related. Detailed is the
antonym of the word.
Hence, [1].

IM

Foreboding

5)

Stupendous

Explanation:

Baleful means harmful or malignant in intent or effect. The word opposite in meaning to it is favourable. Hence, [3].
Question: 186

IMS Learning Resources Pvt.Ltd.,Mumbai.All copyrights to this material vestswith IMS Learning Resources Pvt.Ltd.
No part of this materials either in part oras a whole shall be copied,printed,electronically reproduced,sold or distributed without the written
consent of IMS Learing Resources Pvt.Ltd.and any such violation would entail initiation of suitable legal proceedings.

Copyright

Login ID:1363169/Student Name:SSN/Overall Score:147

Refer to the data below and answer the questions that follow.
Total 10 professionals, named Ajay, Bharat, Chandan, Dinesh, Ehsaan, Farid, Gaurav, Hari, Ishaan and Jay attended a conference. Two
each were from Mumbai, Pune, Nagpur, Nashik and Kolhapur each. They sat around a circular table during lunch time. Following details
about their seating arrangement were known:

1)

Farid

2)

Bharat

3)

Gaurav

4)

Ishaan

5)

Cannot be determined

iv
at
e

Li
m

ite
d

The persons from the same city sat opposite to each other.
Persons from Mumbai were sitting to the immediate right of persons from Pune and to the immediate left of persons from Nagpur.
Persons from Nagpur were not sitting beside persons from Nashik.
Chandan and Dinesh were from Mumbai, Hari and Jay were from Nagpur and Ajay and Ehsaan were from Kolhapur.
Gaurav is equidistant from Ishaan and Dinesh and also to the right of Dinesh and left of Chandan. Jay is to the immediate right of
Dinesh and equidistant from Farid and Bharat.
Which of the following members was definitely from Pune?

IM

Le

ar

ni

ng

es

ou

rc

es

Pr

Explanation:

IMS Learning Resources Pvt.Ltd.,Mumbai.All copyrights to this material vestswith IMS Learning Resources Pvt.Ltd.
No part of this materials either in part oras a whole shall be copied,printed,electronically reproduced,sold or distributed without the written
consent of IMS Learing Resources Pvt.Ltd.and any such violation would entail initiation of suitable legal proceedings.

Copyright

ng

es

ou

rc

es

Pr

iv
at
e

Li
m

ite
d

Login ID:1363169/Student Name:SSN/Overall Score:147

ar

ni

Now all the questions can be answered.


Hence, [3].

Le

Question: 187
Which of the following members were definitely not sitting next to each other?
Chandan and Hari

2)

Gaurav and Bharat

3)

1)

4)

Ehsaan and Dinesh

5)

None of these

IM

Ajay and Ishaan

Explanation:

IMS Learning Resources Pvt.Ltd.,Mumbai.All copyrights to this material vestswith IMS Learning Resources Pvt.Ltd.
No part of this materials either in part oras a whole shall be copied,printed,electronically reproduced,sold or distributed without the written
consent of IMS Learing Resources Pvt.Ltd.and any such violation would entail initiation of suitable legal proceedings.

Copyright

ni

ng

es

ou

rc

es

Pr

iv
at
e

Li
m

ite
d

Login ID:1363169/Student Name:SSN/Overall Score:147

Le

ar

Now all the questions can be answered.

Hence, [4].

IM

Question: 188
If Farid, Dinesh and Jay were sitting together, then which of the following members could have been sitting beside Ehsaan?
1)

Jay and Ishaan

2)

Hari and Ishaan

3)

Jay and Chandan

4)

Hari and Bharat

5)

All of these

IMS Learning Resources Pvt.Ltd.,Mumbai.All copyrights to this material vestswith IMS Learning Resources Pvt.Ltd.
No part of this materials either in part oras a whole shall be copied,printed,electronically reproduced,sold or distributed without the written
consent of IMS Learing Resources Pvt.Ltd.and any such violation would entail initiation of suitable legal proceedings.

Copyright

Login ID:1363169/Student Name:SSN/Overall Score:147

ni

ng

es

ou

rc

es

Pr

iv
at
e

Li
m

ite
d

Explanation:

Le

ar

Now all the questions can be answered.

IM

From the given option, only Hari and Ishaan could have been sitting beside Eshaan.
Hence, [2].
Question: 189
Who was sitting to the immediate left of F? (Use data from the previous question.)
1)

Bharat

2)

Ajay

3)

Ehsaan

IMS Learning Resources Pvt.Ltd.,Mumbai.All copyrights to this material vestswith IMS Learning Resources Pvt.Ltd.
No part of this materials either in part oras a whole shall be copied,printed,electronically reproduced,sold or distributed without the written
consent of IMS Learing Resources Pvt.Ltd.and any such violation would entail initiation of suitable legal proceedings.

Copyright

Login ID:1363169/Student Name:SSN/Overall Score:147

4)

Ishaan

5)

Cannot be determined

Explanation:
Professionals were Ajay(A), Bharat(B), Chandan(C), Dinesh(D), Ehsaan(E), Farid(F), Gaurav(G), Hari(H), Ishaan(I) and Jay(J). They were
from Mumbai(Mum), Pune, Nagpur(Nag), Nashik(Nash) and Kolhapur(Kol).

IM

Le

ar

ni

ng

es

ou

rc

es

From condition 4 and 5, Gaurav was sitting to the immediate left of Chandan.

Pr

iv
at
e

Li
m

ite
d

From condition 1, 2 and 3 we have the following arrangements:

IMS Learning Resources Pvt.Ltd.,Mumbai.All copyrights to this material vestswith IMS Learning Resources Pvt.Ltd.
No part of this materials either in part oras a whole shall be copied,printed,electronically reproduced,sold or distributed without the written
consent of IMS Learing Resources Pvt.Ltd.and any such violation would entail initiation of suitable legal proceedings.

Copyright

ni

ng

es

ou

rc

es

Pr

iv
at
e

Li
m

ite
d

Login ID:1363169/Student Name:SSN/Overall Score:147

Le

ar

Now all the questions can be answered.

IM

From the previous question, it can be seen that Farid was from Pune and Bharat was from Nashik(Nash). Hence, Ishaan(I) was sitting to
the immediate left of Farid(F).
Hence, [4].
Question: 190
Who was sitting three places to the right of Chandan?
1)

Ishaan

2)

Ajay

3)

Ehsaan

IMS Learning Resources Pvt.Ltd.,Mumbai.All copyrights to this material vestswith IMS Learning Resources Pvt.Ltd.
No part of this materials either in part oras a whole shall be copied,printed,electronically reproduced,sold or distributed without the written
consent of IMS Learing Resources Pvt.Ltd.and any such violation would entail initiation of suitable legal proceedings.

Copyright

Login ID:1363169/Student Name:SSN/Overall Score:147

4)

Either Ajay or Ehsaan

5)

Either Ishaan or Ajay

Explanation:
Professionals were Ajay(A), Bharat(B), Chandan(C), Dinesh(D), Ehsaan(E), Farid(F), Gaurav(G), Hari(H), Ishaan(I) and Jay(J). They were
from Mumbai(Mum), Pune, Nagpur(Nag), Nashik(Nash) and Kolhapur(Kol).

IM

Le

ar

ni

ng

es

ou

rc

es

From condition 4 and 5, Gaurav was sitting to the immediate left of Chandan.

Pr

iv
at
e

Li
m

ite
d

From condition 1, 2 and 3 we have the following arrangements:

IMS Learning Resources Pvt.Ltd.,Mumbai.All copyrights to this material vestswith IMS Learning Resources Pvt.Ltd.
No part of this materials either in part oras a whole shall be copied,printed,electronically reproduced,sold or distributed without the written
consent of IMS Learing Resources Pvt.Ltd.and any such violation would entail initiation of suitable legal proceedings.

Copyright

ni

ng

es

ou

rc

es

Pr

iv
at
e

Li
m

ite
d

Login ID:1363169/Student Name:SSN/Overall Score:147

Le

ar

Now all the questions can be answered.

Hence, [1].

IM

Question: 191
Each question has some statements followed by a set of conclusions. Choose the conclusions that logically follow from the
given statements.
Statements:
Some bed-sheets are soft. Some towels are blue. All blue are soft.
Conclusions:
1)

Some bed-sheets are blue.

2)

No towels are bed-sheets.

IMS Learning Resources Pvt.Ltd.,Mumbai.All copyrights to this material vestswith IMS Learning Resources Pvt.Ltd.
No part of this materials either in part oras a whole shall be copied,printed,electronically reproduced,sold or distributed without the written
consent of IMS Learing Resources Pvt.Ltd.and any such violation would entail initiation of suitable legal proceedings.

Copyright

Login ID:1363169/Student Name:SSN/Overall Score:147

3)

Both 1 and 2

4)

Either 1 and 2

5)

Neither 1 nor 2

Explanation:

rc

es

Pr

iv
at
e

Li
m

ite
d

Refer to the following diagram:

Some smart are tall.

3)

Both 1 and 2

4)

Either 1 or 2

5)

Neither 1 nor 2

es

2)

Amar may be smart.

ar

ni

ng

1)

ou

Question: 192
Statements:
Amar and Akbar are tall. Akbar is also smart.
Conclusions:

IM

Le

Explanation:

IMS Learning Resources Pvt.Ltd.,Mumbai.All copyrights to this material vestswith IMS Learning Resources Pvt.Ltd.
No part of this materials either in part oras a whole shall be copied,printed,electronically reproduced,sold or distributed without the written
consent of IMS Learing Resources Pvt.Ltd.and any such violation would entail initiation of suitable legal proceedings.

Copyright

iv
at
e

Li
m

ite
d

Login ID:1363169/Student Name:SSN/Overall Score:147

Some Tamil-speakers are Kannadigas.

2)

Some Tamil-speakers may be Chennaites.

3)

Both 1 and 2

4)

Either 1 or 2

5)

Neither 1 nor 2

es

ou

rc

1)

es

Pr

Question: 193
Statements:
Some Chennaites are Kannadigas. All Kannadigas are Kannada-speakers. Some Kannada-speakers are also Tamil-speakers.
Conclusions:

IM

Le

ar

ni

ng

Explanation:

Some Tamil-speakers may be Kannadigas, but this is not always true as seen in the Tamil-speakers 1 case. Hence, [1] cannot follow.
From the two Tamil-speakers cases, it can be concluded that Some Tamil-speakers may be Chennaites. Thus, [2] can follow. Hence, [2].
Question: 194
Each question has some statements followed by a set of conclusions. Choose the conclusions that logically follow from the
given statements.

IMS Learning Resources Pvt.Ltd.,Mumbai.All copyrights to this material vestswith IMS Learning Resources Pvt.Ltd.
No part of this materials either in part oras a whole shall be copied,printed,electronically reproduced,sold or distributed without the written
consent of IMS Learing Resources Pvt.Ltd.and any such violation would entail initiation of suitable legal proceedings.

Copyright

Login ID:1363169/Student Name:SSN/Overall Score:147

1 and 2

2)

2 and 3

3)

Only 2

4)

4 and 1

5)

Only 1

Li
m

1)

ite
d

Statements:
Some trousers are capris. Some capris are red. All trousers are blue garments.
Conclusions:
Conclusion 1: Some blue garments are red.
Conclusion 2: Some capris are blue garments.
Conclusion 3: Some trousers are red.
Conclusion 4: No capris are blue garments.

es

ou

rc

es

Pr

iv
at
e

Explanation:

ng

Thus, conclusion 1 cannot always follow. As some trousers are capris and all trousers are blue garments, conclusion 2 can follow. Since
conclusion 4 contradicts conclusion 2, it has to be incorrect. Conclusion 3 cannot always follow. Hence, only conclusion 2 can follow.
Hence, [3].

1 and 3

IM

1)

Le

ar

ni

Question: 195
Statements:
Some teenagers are naughty. All girls are studious. Some boys and girls are naughty.
Conclusions:
Conclusion 1: Some teenagers are studious.
Conclusion 2: No boys are studious.
Conclusion 3: Some boys may be teenagers.
Conclusion 4: No studious is naughty.

2)

2 and 3

3)

Only 1

4)

1 and 2

5)

Only 3

Explanation:

IMS Learning Resources Pvt.Ltd.,Mumbai.All copyrights to this material vestswith IMS Learning Resources Pvt.Ltd.
No part of this materials either in part oras a whole shall be copied,printed,electronically reproduced,sold or distributed without the written
consent of IMS Learing Resources Pvt.Ltd.and any such violation would entail initiation of suitable legal proceedings.

Copyright

iv
at
e

Li
m

ite
d

Login ID:1363169/Student Name:SSN/Overall Score:147

Conclusion 1 will not always follow, as can be seen from the case Teenagers 1. As seen in the case Boys 1, conclusion 2 may not
always follow either. Conclusion 3 may follow as can be seen from the various Boys and Teenagers cases. Conclusion 4 contradicts the
result of the third and fourth statements. Thus, only conclusion 3 follows. Hence, [5].

60%

3)

75%

4)

64%

5)

56%

rc

2)

ou

80%

es

1)

es

Pr

Question: 196
Choose the correct alternative.
A retailer gives 3 successive discounts of 10%, 20% and 50% on marked price of an article. What will be the effective rate of discount??

IM

Le

ar

ni

ng

Explanation:

Question: 197
In which of the following options is the simple interest earned different from that in the other options?
1)

P = Rs. 1000; R = 6%; N = 4

IMS Learning Resources Pvt.Ltd.,Mumbai.All copyrights to this material vestswith IMS Learning Resources Pvt.Ltd.
No part of this materials either in part oras a whole shall be copied,printed,electronically reproduced,sold or distributed without the written
consent of IMS Learing Resources Pvt.Ltd.and any such violation would entail initiation of suitable legal proceedings.

Copyright

Login ID:1363169/Student Name:SSN/Overall Score:147

2)

P = Rs. 2000; R = 3%; N = 4

3)

P = Rs. 500; R = 4%; N = 12

4)

P = Rs. 800; R = 6%; N = 4

5)

P = Rs. 4000; R = 3%; N = 2

es

Pr

iv
at
e

Li
m

ite
d

Explanation:

3)

4)

5)

10

ou

2)

es

1)

rc

Question: 198
If nP3 = 504, then calculate the value of n.

Le

ar

ni

ng

Explanation:

IM

Question: 199
A bus travels the first 150 km of a journey at 50 km/hr, next 225 km at 25 km/hr and the final 240 km at 80 km/hr. Find the average speed
of the bus for the entire journey.
1)

35 km/hr

2)

37 km/hr

3)

39 km/hr

4)

41 km/hr

5)

43 km/hr

IMS Learning Resources Pvt.Ltd.,Mumbai.All copyrights to this material vestswith IMS Learning Resources Pvt.Ltd.
No part of this materials either in part oras a whole shall be copied,printed,electronically reproduced,sold or distributed without the written
consent of IMS Learing Resources Pvt.Ltd.and any such violation would entail initiation of suitable legal proceedings.

Copyright

Login ID:1363169/Student Name:SSN/Overall Score:147

ite
d

Explanation:

45

2)

50

3)

55

4)

60

5)

65

iv
at
e

1)

Li
m

Question: 200
A batsman scored at an average of 40 runs in first 10 matches and scored exactly 100 runs in each of the next 5 matches. What will be
his average in the 15 matches?

IM

Le

ar

ni

ng

es

ou

rc

es

Pr

Explanation:

IMS Learning Resources Pvt.Ltd.,Mumbai.All copyrights to this material vestswith IMS Learning Resources Pvt.Ltd.
No part of this materials either in part oras a whole shall be copied,printed,electronically reproduced,sold or distributed without the written
consent of IMS Learing Resources Pvt.Ltd.and any such violation would entail initiation of suitable legal proceedings.

Copyright

Das könnte Ihnen auch gefallen